На КПВД картина "Planets" Alexa Pi
На КПВД картина "Planets" Alexa Pi

Знаете, что самое неприятное в научных спорах? Вот ты беседуешь, о какой-то космологической концепции, а оппонент берёт и приводит довод: «А вот в фантастическом романе…»

И ты такой: «А… ну… да…». И быстренько сворачиваешь дискуссию. Потому что художественная литература очень сильно отличается от реальности. Это я вам как литератор говорю. Персонажи и события в книгах происходят не по воле случая, как в реальной жизни, а следуют воле творца — который составляет из них историю или иллюстрирует мысль. Даже к точным и честным мемуарам нужно относиться с настороженностью, из-за гнездящейся в них «систематической ошибки выжившего».

Это необходимый дисклеймер я написал потому, что собираюсь критически переосмыслить концепцию «Темного Леса» Лю Цысиня. Не покритиковать, нет — сама по себе эта концепция прекрасна. Если принимать её как метафору взаимодействия власти и оппозиции. Тоталитаризм так работает — если ты начинаешь мыслить и обретаешь голос, то ты исчезаешь. В лучшем случае тебя просто удаляют из информационного пространства. В худшем… не будем о худшем. Эта статья не о «Темном лесе» как метафоре.

Я предлагаю рассмотреть «Темный лес» с точки зрения теории игр. Конечно, в определенном смысле, это тоже будет не научная, а литературная интерпретация. Просто потому, что точных научных данных пока нет, а «Similia similibus curentur» (подобное лечится подобным).

Вот как сформулировал концепцию Темного Леса сам автор:

Вселенная — это темный лес. Каждая цивилизация — вооруженный до зубов охотник, призраком скользящий между деревьев, незаметно отводящий в сторону ветви и старающийся ступать бесшумно. Он даже дышит через раз. Охотнику есть чего опасаться: лес полон других невидимых охотников, таких же, как он сам.

Если он встретит жизнь — другого охотника, ангела или черта, новорожденного младенца или старую развалину, фею или полубога, — у него лишь один выход: открыть огонь и уничтожить.

В этом лесу другие люди — ад. Любая жизнь представляет собой смертельную угрозу для всех остальных и будет уничтожена при первой возможности. Вот так выглядит космическая цивилизация. И этим объясняется парадокс Ферми.

(с) Темный лес (пер. Д. Накамура)

Переведем этот художественный образ в более строгую форму. Итак: Имеется вероятность, что другая форма жизни уничтожит тебя. Поэтому самый безопасный вариант – уничтожить другую форму жизни, прежде чем она уничтожит тебя.

 По моему скромному мнению, эта концепция страдает неполнотой. В ней опущены многие важные факторы. Например, лес, в котором стреляют — вовсе не такой темный. В концепции Лю Цысиня подразумевается, что схватки между цивилизациями всегда происходят один на один. Есть охотник. Есть жертва. И всё.

Но в лесу полным полно других цивилизаций разной степени развития. И все они жадно вглядываются в космос, пытаясь, как и мы, найти ответы на главные вопросы: «Кто мы?», «Откуда мы взялись?», «Что нас ждет?» и «Где все остальные?»

Во вселенной «Темного леса» космос не был бы безмолвной бездной. Просто потому, что развитые космические цивилизации — они как борщевик. В смысле, их очень сложно уничтожить под корень. Всегда остаются отдельные ростки: форпосты, исследовательские группы, колонии… Часть из них спрячется, стремясь сохранить свой род. А часть — займется уничтожением напавшего на них врага.

Единственный доступный им способ причинить вред многократно более сильному врагу — это создать союз. Объединить будущие жертвы. Потому что цивилизация, живущая по принципу темного леса — заведомый враг всех разумных рас. А враг моего врага — мой друг.

Первый шаг к устройству такого союза — оповещение потенциальных жертв об охотнике. Этот принцип настолько всеобъемлющ, что присутствует даже у неразумных существ. Да, я о криках испуга или боли, издаваемых особями, схваченными хищником. Саму жертву они не спасают, но помогают выжить другим членам стаи, закрепляя, таким образом, этот паттерн через эволюционный отбор.

Так что существуй темный лес в реальности — мы давно бы получили предупреждения от братьев по разуму. А поскольку их нет — а их нет, значит, молчание вселенной объясняется каким-то другим способом.

Например, наличием лесника.

Кто такой лесник? Поскольку сегодня мы рассматриваем литературные образы, то представляю вам классический анекдот, известный во многих интерпретациях. (Я взял первую попавшуюся)

Дневник Василия Ивановича:

День первый: Крутой кавалеристской атакой мы выбили белых из леса.
День второй: Белые подтянули свежие войска и выбили нас из леса.
День третий: К нам подошли тачанки с пулеметами и выбили белых из леса
День четвертый: Белые подтянули бронепоезд и выбили нас из леса.
День пятый: Пришел лесник и разогнал всех к черту.

Понимаете, к чему я клоню? Нет? Тогда придется объяснить всё с начала. С момента большого взрыва. Для начала напомню вводные. Наше Солнце — звезда третьего поколения. Множество звезд родились, сияли и взорвались задолго до его появления. Сложившие на Земле условия, приведшие к появлению жизни, так же нельзя назвать уникальными. Сложив эти факторы, мы приходим к выводу, что человечество далеко не первая возникшая в космосе цивилизация. Жизнь и разум возникли задолго до нашего появления.

Давайте рассмотрим первую генерацию разумных существ, повсеместно проявляющихся на разных планетах на самой заре времен — у звезд второго поколения, как только условия универсума стали пригодны для возникновения жизни.

Раз возникнув, цивилизации развивались, осваивая близлежащие звездные системы. Неизбежно наступило время контактов — когда зоны экспансии разных цивилизаций стали соприкасаться. Ни одна из этих звездных империй не имела подавляющего превосходства — одновременный старт диктует примерно равные технические возможности.

Попробуем предсказать итог этих контактов. Немного упростив задачу, представим что заселяющие космос цивилизации могут быть двух видов: злые и добрые. Злые — стараются уничтожить всех соседей, добрые — заключить с соседями союзы.

Участь злых цивилизаций незавидна. Начав с геноцида, они быстро сталкиваются с множеством проблем — потратив ресурсы на уничтожение соседей, они ничего не получают взамен. Кроме жизненного пространства — которого в космосе и без этого много больше чем требуется. При этом их соседи, добрые цивилизации, без проблем заключают союзы, противопоставляя злыдням объединенную мощь.

Проиграв несколько сценариев взаимодействия цивилизаций, я пришел к выводу, что зло всегда уничтожает само себя. Довольно банальный вывод, постоянно воспроизводящий себя в сотнях конфликтов по всему миру. Появление прямого и явного злодея Гитлера привело к союзу двух непримиримых конкурентов: СССР и США, которые после победы над злом организуют ООН. Конечно, дальнейшее развитие событий далеко от сусального «...и жили они часто и счастливо», но много лучше, чем скроенный по лекалам Гитлера мир.

Поскольку космические цивилизации, в отличие от Гитлера, могут достоверно предсказывать последствия своих действий, можно предположить, что они выберут вариант сотрудничества, организовавшись в федерацию. Конечно, это предположение, обоснованно опирающее на пример единственной известной нам высокоразвитой цивилизации.

Нашей цивилизации. Опираясь на наш собственный культурный опыт мы можем предсказать черты этой гипотетической «Звездной федерации». Давайте вернемся к концепции «Темного леса». Крайне маловероятно, чтоб союз выживших (выживших всех злодеев) миролюбивых рас будет заниматься планомерным геноцидом.

Лирическое отступление: Мой коллега Иван, с которым мы обсуждали эту концепцию, на этом месте возмущенно воскликнул:

— Высокоразвитые космические сообщества будут руководствоваться не эмоциями, а логикой! Которая диктует, что нужно уничтожить всех потенциальных конкурентов, чтоб не иметь с ними проблем в дальнейшем.

— А ты бы не мог подтвердить этот тезис личным примером? Вот, возьми чайник с кипятком и залей муравейник под яблоней. (Мы сидели на дачной веранде). Этим ты обезопасишь нас от возможного вторжения муравьев на кухню.

— А вот пойду и залью, — вскочил Иван, — и докажу тебе, Фоме неверующему, что высшему разуму плевать на низшие формы жизни!

Не было его минут пять. За это время я успел съесть все свои оладушки и начать прицениваться к остывающим на тарелке чужим. Но немного не успел — меня отвлек Иван, со стуком поставивший обратно на стол полный чайник.

— Ну, как успехи с геноцидом? — спросил я.

— Они суетятся… — пробормотал Ваня, — бегают. Ящерицу дохлую в муравейник тащат. Пойду им сахару дам, — добавил он, вставая.

 Мораль истории, думаю, очевидна. Но её можно подкрепить и логическим доводами — давайте рассмотрим модель взаимоотношения Федерации с неприсоединившимися планетами. Основной моделью, я думаю, будет невмешательство и нейтралитет. Пользы от подрастающих цивилизаций немного, а обучать их, подтягивая до своего уровня — только портить. В этом мы тоже успели убедиться.

Единственная возможная причина, по которой союз старейших сверхцивилизаций начнет вмешиваться в процесс взросления цивилизаций — это планомерный геноцид соседей. Агрессивные захватчики обязательно станут врагами союза. Почему? Потому что «Паровозы нужно уничтожать пока они чайники!»

Эту литературную концепцию я приводить уже не буду, поскольку три литературные концепции на статью — это уже перебор. Я просто подведу итог: Если в темном лесу тихо — значит за порядком следит лесник.

И это — хорошо.

Сцена после титров:

Одним из самых популярных вопросов, которые задавали мне во время обсуждения этой концепции, является вопрос экспансии. Если я постулирую появление разумной жизни на самой заре вселенной, то почему этот разум не расширился многократно, заполнив собой все имеющееся жизненное пространство?

Давайте рассмотрим этот случай на упрощенной модели — игре «Жизнь». Математик Джон Конвей придумал эту игру в 1970 году. По сути, это набор предельно простых правил, благодаря которым без всякого компьютера, просто на доске с фишками, можно изучить общие принципы существования всего живого, потому, что физические законы едины и для простенькой модельки и для межзвездных империй. 

Для начала создадим универсум. Итак: вселенная была безвидна и пуста. И в ней, там и тут, повинуясь встроенным в неё правилам, стала зарождаться жизнь.

Наша задача повторить это в модели: Возьмем огромную шахматную доску (это мысленный эксперимент) на которую высыплем фишки — без логики и всякого порядка. Каждая фишка – это живая клетка. Распределение живых клеток в начале игры мы называем первым поколением. Как и всё в мире, эти живые клетки, подчиняются физическим законам. В нашей упрощенной модели их всего два: 

  • В пустой (мёртвой) клетке, рядом с которой ровно три живые клетки, зарождается жизнь. 

  • Если у живой клетки есть две или три живые соседки, то эта клетка продолжает жить; в противном случае (если соседей меньше двух или больше трёх) эта клетка умрет. 

Пока всё понятно? Тогда давайте запустим в нашей вселенной время! Применим правила к хаотично разбросанным фишкам. Уберем все умершие от одиночества или перенаселения клетки. На засеянной доске сразу стало свободнее.  И это позволяет нам понять один из законов вселенной:

Жизнь во вселенной легко возникает, но также легко и погибает. Только единицы, могут, по чистой случайности, сложиться в устойчивую конфигурацию (Обрести разум дающий устойчивость против гибельных изменений окружающей среды).

Цивилизации «Блок» и «Мигалка» — в рамках игры будут существовать вечно. А ценой, которую они заплатили за вечную жизнь — будет отказ от экспансии. Если этот вывод не кажется вам очевидным — то давайте рассмотрим пример развития вот этой вот конфигурации: R-пентамино.

Вы можете наблюдать за жизнью и смертью галактической империи в миниатюре. После долгого периода экспансии, когда империя занимает огромную, по сравнению со стартовой площадь, наступает спад, в результате которого от империи остаются множество стабильных, неизменных обломков и лишь несколько малых областей, в которых все еще теплится жизнь.

Захватывающее зрелище, правда? Позволяющее нам сделать вывод: «У бурных чувств неистовый конец, Он совпадает с мнимой их победой. Так сладок мед, что, наконец, и гадок... Лишь в чувстве меры истинное благо…»

То есть конечно, «Экспансия не может длиться вечно» — если избиваться от литературности. Это и есть ответ на наш вопрос — именно по этой причине вселенную на заре времен, не заняла одна из молодых да ранних рас.

 

Комментарии (228)


  1. Goupil
    01.08.2021 15:22
    +7

    Все эукариоты ( существа с ядром), к которым относимся и мы, суть союз двух-трёх изначально совершенно независимых существ ( бактерий и археев). Бактериальные сообщества - сложнейшие социумы, где есть место и эгоизму, и альтруизму, и сотрудничеству. Не склонные к симбиозу охотники никуда не ушли. В одиночестве и злобе не прожить.


  1. iliasam
    01.08.2021 15:39
    +1

    Если не ошибаюсь, в книге важным фактором были огромные расстояния между звездами и невозможность двигаться и передавать данные быстрее скорости света, из-за чего межзвездные союзы получались невозможными.
    Плюс в книге упоминается страх цивилизаций по проводу «технологического взрыва» у соседей, который может внезапно превратить «слабую» цивилизацию в «сильную».


    1. Zangasta Автор
      01.08.2021 15:46

      Я не спорю, что в книге это обоснованно и работает. Кольцо всевластья в романе всесильно, верно? Вопросы возникают когда эту концепцию переносят из романа в реальность.


    1. T_Cirkla
      01.08.2021 21:26
      +1

      Есть таки мнение, что такие цивилизации, которые могут решить вооружаться и на космическом, звёздном, уровне, банально не дойдут до этого уровня, ибо успеют уничтожить себя до объединения в рамках своей планеты.


      1. Moretti2020
        02.08.2021 13:51
        +1

        По такой логике и земные государства, имеющие собственную армию, не дойдут до этого уровня и успеют уничтожить себя ещё в общинно-племенной период. Но нет, они существуют. «Тёмный лес» ближе, чем нам кажется.


      1. vikarti
        03.08.2021 14:41

        Закон Синед Роба?
        Встречался мне как то фанфик где попаданец в мир где оный закон считается действующим и доказательства есть, начинает изучать историю на тему чем этот мир отличается от его родного. И всплывает ситуация что да — закон официально считается действующим но при том повоевать в космосе эта версия Земли успела (только чудом не дошло до открытого боя примерно равных сил, а не — равные боевые действия — были, пусть и не на полное уничтожения).


    1. AlexSkvortsov
      02.08.2021 11:06
      +2

      Вот только... в той же книге есть Софоны, которые хоть первоначально и распространяются со световой скоростью, в дальнейшем информацию передают мгновенно)


    1. vikarti
      03.08.2021 14:36

      Что мешает иметь Великое Кольцо и передавать по радио?
      Вообще в рамках литературных концепций… вспоминается вот Валерий Быков с СИ -:). Там с одной стороны именно что подход что другие цивилизации — угроза. Альянсы — проблема. Но вот вывод чуть другой, новые цивилизации совершенно не обязательно уничтожать, можно ассимилировать. При это важна скорее культура и ситуация когда суперцивилизацию задавили но кто то нашел артефакты вообще предусмотрена(!), культура и техносфера будут возрождены на базе другого вида, даже если это не очень совместимо с культурой этого иного вида.


  1. Nalivai
    01.08.2021 15:59
    +3

    Чего-то у вас все формы жизни не просто земноподобные, а прям человекообразные какие-то, с логикой человеческих землян,
    чистый стар трек.


    1. Zangasta Автор
      01.08.2021 16:02
      -5

      Просто потому, что никакой "другой логики" нет --- два умножить на два будет четыре абсолютно у всех разумных рас.


      1. Nalivai
        01.08.2021 16:09
        +2

        А два умножить на два будет "что-то мне муравьишек жалко" тоже у всех?


        1. vvzvlad
          01.08.2021 16:17
          +3

          «А вы едите своих детей?»


          1. thewall
            01.08.2021 21:17
            +7

            «А вы едите своих детей?»

            Своих нет, только чужих: домашний скот, птица, морепродукты. И это считается нормальным у людей. Разве нет? Точно так же среди пришельцев будет нормальным пострелять в вон тех забавных человечков на ихнем «сафари».


            1. Zangasta Автор
              01.08.2021 21:43

              Когда я был маленьким, я читал Майн Рида. У которого в некоторых романах просто бойня животных --- он подробно описывал сафари, на которых кровь лилась рекой.

              Мне уже тогда это было неприятно читать. Мораль изменилась. А у современного читателя эти сцены вызывают отвращение.

              Так что сафари --- отомрет. Пока что им развлекаются толстосумы, именно затем, чтоб доказать общественная мораль им не писана. Но и это отмирает.


            1. vikarti
              03.08.2021 14:47

              У Юдковского есть рассказ Тройной контакт. У одной из цивилизаций ответ — да, как же иначе.
              Две другие слегка фигеют.
              Друг от друга — тоже.
              По уровню развития все три цивилизации почти равны.
              В одном из вариантов одна из этих двух цивилизаций (не самая развитая) приняла меры чтобы закрыть контакт потому что опасались последствий.


          1. Ndochp
            03.08.2021 16:52
            +1

            В на Тройной контакт намекаете?
            https://royallib.com/read/yudkovski_eliezer/troynoy_kontakt.html#0


        1. Zangasta Автор
          01.08.2021 16:22
          -2

          Я думаю что у всех. Не уважая ценность чужой жизни сложно сосуществовать с соседями. Поэтому все выжившие так или иначе жизнь ценят.


          1. kogemrka
            02.08.2021 13:16
            +2

            Свиней забивают на мясо уже не одну тысячу лет.

            Исторические данные, которые мы имеем - каждая тысяча лет, на протяжении которой свиней забивали на мясо, в последствии оказывалась тысячей лет на которой мы как вид выжили и с соседями успешно просуществовали.

            Флуктуация в морали, о которой вы говорите в соседнем комментарии (цитирую: "Мне уже тогда это было неприятно читать. Мораль изменилась. А у современного читателя эти сцены вызывают отвращение.") - даже один век не просуществовали.

            К слову, как раз в этот век человечество создало оружие судного дня и обеспечило гарантии максимально жёсткого взаимного уничтожения себя вместе с соседями.


            1. WASD1
              02.08.2021 13:40

              >> Исторические данные, которые мы имеем... оказывается тысячей лет на которой мы выжили.

              Это прям классическая ошибка выжившего. Если бы в очередную тысячу лет человечество, как цивилизация, не выжила - некому было бы рассматривать "наши исторические данные".


              1. kogemrka
                02.08.2021 14:32

                Конечно) Даже смотреть на прошлые тысячи лет - не самая хорошая идея)

                А уж ориентироваться исключительно на нынешний век, исход которого нам в принципе не известен - вообще как-то странно.


            1. torbasow
              04.08.2021 12:15

              Это не флуктуация, это тенденция. Которая не в последние сто лет началась. Предыдущими шагами были отказы от людоедства и рабства.


              1. kogemrka
                04.08.2021 13:22

                Дык, свиней забивать начали не то что в последние сто лет - но даже не в последнюю тысячу)

                И потребление мяса в килограммах на душу населения, насколько мне известно, только растёт. Это тенденция. В циферках наблюдаемая.


                1. torbasow
                  05.08.2021 06:51

                  Неправильно Вам известно. Я вот смотрю статистику, и вижу, например, в Канаде, явную и сильную тенденцию на сокращение: 108 кг/чел. (2002 г.) → 94 (2009) → 70 (2017). И по очень-очень многим странам та же картина, рост (как в Южной Корее) — скорее исключение.
                  Да, там указано, что с этими данными не всё так просто. Другие источники (например) показывают, что потребление мяса росло в бедных странах, в то время как в богатых оставалось на прежнем уровне. Это, однако, тоже не позволяет утверждать, что существует универсальная и неостановимая тенденция.
                  И потом, мясо мясу рознь. ФАО утверждает, что в 1990—2009 годах значительно возросло подушевое потребление птицы, гораздо менее значительно — свинины, а потребление говядины сократилось.
                  Посмотрим, что будет с этим дальше. Между тем, культурная тенденция ограничения и сокращения потребления мяса явно развивается, и пока не видно, что может ей помешать возобладать в конце концов.


    1. T_Cirkla
      01.08.2021 21:24

      Так стар трек, показывая инопланетные цивилизации, на самом деле и показывает через них разные стороны человечества.


  1. mad_god
    01.08.2021 16:18
    +2

    Давайте поступим проще. Не будем брать во внимание далёкие и непонятные инопланетянские расы, которые где-то там сидят и слушают эфир, и могут лететь к нам тысячи лет.

    Давайте представим, допустим, наше человечество через тысячу лет.

    Такое себе предсингулярное человечество, одной ногой стоящее в новой технологической революции.

    Уровень жизни, допустим, у всех желающих работать и развиваться, довольно неплохой. При желании можно заработать на мобильник, компьютер, или машинное время на хорошем мейнфрейме, накопить достаточно денег, чтобы заказать какие-то особенные комплектующие, приборы, заказать микросхему, печатную плату, достать особую линзу или химическое вещество.

    Есть какой-нибудь гараж, есть интерес что-то изобретать, интерес читать научные статьи, общаться с такими же гиками и так далее.

    В совершенно разных странах, в разных таких гаражах люди изучают что-то своё, какую-то новую область физики или информатики или чего-то ещё. Собирают новый материал при низких температурах и высоком давлении или что-то подобное.

    Изобретения сыпятся как из ведра. Один изобретатель создаёт материал, который покрывает тело тонкой плёнкой и его нельзя пробить ни одним огнестрельным и холодным оружием.

    Другой изобретатель создаёт компактный источник энергии, в котором фотоны заключены в сложную топологическую ловушку, из которой выбираться будет тысячу лет, но создать которую возможно в короткое время при помощи одновременной работы миллионов управляемых роботов. Технология очень хорошо масштабируемая.

    Третий создаёт прибор, собирающий предметы любой сложности и сочетаний материалов из кварковой плазмы, которую научился довольно легко добывать из любой окружающей массы.

    Неизвестно где и неизвестно когда разные люди становятся обладателями технологий, сравнимых сейчас с магией, и которые дают своим владельцам большой буст к возможностям, к влиянию на других людей, на их безопасность, на экономику целых стран.

    Эти люди становятся целью для текущих властей, которые этими технологиями не обладают, хотят ими завладеть, взять их под контроль, отобрать и закрыть их от простых людей.

    Каждый такой человек может стать кем-то вроде бога или полубога, обладающего особыми силами, молниями, плазмой, кинетической энергии, гравитации, магнетизма и такого прочего.

    Это не обычный прохожий, которого вы можете толкнуть в плечо в толпе или приказать ему бросить оружие. Он взмахнёт рукой и сотня человек полетит вправо, взмахнёт второй, сотня полетит влево, вместе с машинами. Щёлкнет пальцами и вы забудете кто вы и куда шли. Подберёт определённые слова и вот вы уже полностью ему доверяете и готовы за него умереть.

    Каждый хороший студент может собрать у себя в гараже летающую машину, которая унесёт его на Луну, точно так же как сейчас можно собрать какое-то радио, что совсем недавно было верхом технологий.

    Как какой-то обычный гражданин или представители власти смогут ограничить разум, любопытство, энтузиазм?

    Как одно древнее племя могло помешать другому изобрести огонь, лук, колесо? Когда обладание этими технологиями становило изобретателей на ступени выше других. От них зависело, будет ли существовать другое племя или нет, если они применят эту технологию против них.


    1. Zangasta Автор
      01.08.2021 16:26
      -2

      Это уже больше магия, чем реальный прогноз. Сингулярности не будет --- мы выходим на плато и каждый следующий шаг будет сложнее.


    1. konst90
      01.08.2021 19:00
      -1

      Откуда уверенность, что гаражному самоучке-изобретателю станет возможно превзойти достигнутый крупными корпорациями технологический уровень настолько, чтобы получить индивидуальное могущество?


      1. tvr
        01.08.2021 19:52
        +2

        Откуда уверенность, что гаражному самоучке-изобретателю станет возможно превзойти достигнутый крупными корпорациями технологический уровень

        Возможно он найдёт рояль, забытый в кустах тинейджерами из сверхцивилизации, которые тусили поблизости с его гаражом некоторое время тому назад, и использует его вместо верстака. Т.к. это рояль сверхцивилизации — унутре всякие сверхструны, кардинально меняющие свойства материалов и механизмов обрабатываемых на его антикварной крышке из унобтаниума.


        1. Firz
          01.08.2021 20:25
          -1

          Зачем так усложнять то все — просто найдет потерянный «телефон» таких тинейджеров и закажет доставку(телепортацией) нужной вещи с «алиэкспресс».


          1. tvr
            01.08.2021 20:50
            +2

            с «алиенэкспресс».


    1. T_Cirkla
      01.08.2021 21:27

      Здесь есть один лишний пункт — деньги.


    1. DaneSoul
      01.08.2021 21:42
      +1

      Эти люди становятся целью для текущих властей, которые этими технологиями не обладают, хотят ими завладеть, взять их под контроль, отобрать и закрыть их от простых людей.
      Эти технологии у власти в первую очередь и появятся, потому как над подобным будут работать целые НИИ, противопоставить которым что-то одиночке из гаража малореально.
      Посмотрите на развитие науки — чем дальше, тем сложнее и ресурсозатратней она становится. Это пару веков назад ученый мог делать открытия сразу в физике и химии используя простое оборудование у себя в гараже. Прорывные открытия уже давно делаются в огромных НИИ с кучей специалистов и сложнейшим оборудованием, а не в гаражах.


      1. mad_god
        01.08.2021 23:32

        Есть несколько приоритетных направлений, в которых работают крупные корпорации, это факт. Факт ещё в том, что ни один человек не может предсказать точное время появления следующей прорывной технологии и что это будет.

        Как, например, появление и широкое распространение смартфонов с сенсорным экраном и операционкой Андроид, ведь сколько было совершенно разных мобильников и операционных систем для них, даже типов зарядок.

        Было ли ясно заранее, в каком году данная технология захватит мир?

        Что также повлекло создание кучи мобильных приложений и так далее.

        Можете ли предсказать, что следующее так "выстрелит"? Роботы-помошники? Секс-роботы? Автоматическая доставка товаров? Система пневмопочты?

        В какой области науки и техники будет этот скачок?

        Я, например, считаю, что я давно бы создал ИИ, если бы у меня было бы более устойчивое материальное положение.

        Например, я считаю, что хоть и большие корпорации сделали много важных шагов в развитии ИИ, они пока что пропускают создание сверхминиатюрных функциональных элементов типа "нейрон", которые очень быстро могут включаться и выключаться, чтобы исполнять случайные функции без зависаний. А мне это видится очевидным и важным шагом.

        Без этого сложнее подобрать необходимые логические или математические функции для обработки тех или иных данных, отослать набор данных на обработку, получить ответ, сверить с необходимым тестовым ответом, пометить адрес функции как полезный или бесполезный, пересобрать бесполезную функцию по другому и так далее.

        Одни функции можно связать с камерами, другие - с микрофонами, третьи с сервоприводами, и так далее.

        Обычные микропроцессоры для этого не годятся, им нужно время, чтобы загрузить систему, стать готовыми для операций, они не могут ответить, если зависли и ничего не могут сами сделать со своим зависанием.

        Таким могло бы быть написание программ самим ИИ, подбор нужных функций, отвечающих требуемым параметрам, передача информации по цепочке этих функций, сворачивающих информацию до адресов функций и последовательностей функций, помеченных набором адресов. Перекоммутация кластеров функций между собой, создание функций-моделей, генерирующих паттерны, соответствующие паттернам, которые наблюдаются у объектов реального мира или у абстракций, моделей, которые возникают при мыслительных операциях, представлении объектов реального мира, воображении новых качеств у известных объектов. И тому подобное.

        Я мог бы быть тем человеком, который создал бы ИИ ещё в 2010 году, например, просчитал бы, каким образом можно было повернуть финансовую систему в свою пользу, создал бы криптовалюту, просчитал бы все последствия этого, создал бы колл-центры без единого живого человека, подталкивал бы разных людей к различным действиям, что-то покупать, куда-то идти работать, учиться и так далее, что принесло бы выгоду сотням моих открытых фирм и фирмочек.


    1. Yuribtr
      02.08.2021 00:41

      1. Мне кажется, или Вы в своих рассуждениях опираетесь на нынешнюю мораль общества? Вы серьезно полагаете что через 1000 лет мораль и приоритеты людей не поменяются?

      2. IMHO самоучки вряд ли будут способны "в гараже" создать прорывную технологию без одновременного создания подобного в других местах мира (как это было с изобретением телефона, радио и прочего), есть такая поговорка что "идеи витают в водухе". Для открытия прорывной технологии должен быть подготовлен плацдарм.


      1. mad_god
        02.08.2021 08:40

        Не знаю, какую роль здесь играет мораль. Вряд ли она станет единой для всех, думаю, отличия то там то здесь дадут возможность существования нужных людей с нужными устремлениями. Даже если весь мир перейдёт в виртуальность, кто-то всё равно будет творить, кто-то будет исследовать и интересоваться знаниями, как всё связано.

        Идеи витают в воздухе. Но возможных технологий так много, много физических феноменов, на которых они основаны. Например, можно будет получить нейтринный лазер, поток самых мельчайших частиц и все бросятся изучать, что с его помощью можно сделать. Возможно, устанавливать каждый атом на нужное место, сжимать их двумя потоками нейтрино воедино, а может, разрывать пространство и перемещать предметы в четвёртом измерении, строить мельчайшие микропроцессоры на совершенно новом техпроцессе, делать совершенно новые материалы, лёгкие, прочные, топологически до этого невозможные.

        Просто сейчас нет такого искусственного источника нейтрино, и никто не занимается этой областью. А этих областей очень много и они разветвляются как огромное дерево.


  1. DesertFlow
    01.08.2021 16:52
    +1

    После долгого периода экспансии, когда империя занимает огромную, по сравнению со стартовой площадь, наступает спад

    Это верно для животного мира. Но разумный вид на то и разумный, чтобы иметь лучшую выживаемость. Поэтому разумная империя скорее всего будет жить вечно, находясь в балансе с доступными ресурсами. А ресурсов в космосе много.


    И есть ещё интересная тема околосветовых путешествий. Разумная цивилизация может отправить ковчеги на близкой к скорости света. Для обитателей корабля пройдут месяцы или годы, пока они летят, а для исходной цивилизации сотни тысяч лет. Даже если такой закон свществует, что любая цивилизация приходит к упадку, эти летящие остаются представителями цивилизации с этапа экспансии. Потом они запустят свои ковчеги, и так далее. В итоге эта цивилизация будет все время оставаться на этапе экспансии, даже если оставленные на планетах колонии будут приходить по этому закону в упадок.


    1. bogolt
      01.08.2021 19:29

      кстати цивилизация Жизни из примера автора запустила штук пять глайдеров в разные стороны


      1. Zangasta Автор
        01.08.2021 19:50
        -4

        Но ни один из этих глайдеров не станет основой новой колонии.

        Они так и будут существовать --- вечно путешествуя сквозь пространство и время к концу вселенной. Встречающиеся им на пути культуры будут называть их "Странниками", евпочя.


        1. bogolt
          01.08.2021 19:55
          +5

          Но если на пути они встретят другую колонию то они могут на нее повлиять, привести к расширению или наоборот к смерти, или просто бесславно погибнуть ни на что не повлияв.


  1. rombell
    01.08.2021 17:09
    +5

    По современным понятиям у звёзд первого поколения просто не из чего было образовывать планеты земного типа, так как всё тяжелее лития появилось при взрывах этих самых звёзд первого поколения. Опять же по современным (уточнение — моим) понятиям жизнь на газовых гигантах типа Юпитера, возможных у первых звёзд, вряд ли выйдет в Космос за отсутствием материалов.
    Так что лесниками могут быть только зародившиеся у звёзд второго поколения. А тут мы не младше всех, по-видимому, так как следов всё ещё не наблюдается.

    UPD Луны у газовых планет тоже требуют твёрдого вещества, которого во времена первых звёзд не было. Где-то мне попадались оценки, что Солнце — одна из ранних звёзд второго поколения, поскольку первые должны были прогореть, коллапсировать, а материал ещё долететь и скучковаться.


    1. Zangasta Автор
      01.08.2021 17:14
      -1

      понятиям жизнь на газовых гигантах типа Юпитера,

      Мне сложно представить себе Юпитер без системы лун. Да и Солнце не самая ранняя звезда второго поколения.


    1. T_Cirkla
      01.08.2021 21:31

      А если планета и звезда, одинаковая по возрасту с нашей планетой, находится на таком расстоянии, что банально сигналы от её цивилизации ещё не достигли нас? А что, если цивилизация слегка моложе, на пару сотен или тысячу лет и у них ещё нет средств для отправки сигналов?


      1. rombell
        02.08.2021 00:06

        Тогда с хорошей точностью можно считать, что эти существа до нас никогда не долетят. Существам за миллионы световых лет от нас явно есть чем заняться и помимо нас.


        1. DesertFlow
          02.08.2021 00:44

          Но они могут послать электромагнитный пакет данных, содержащий инструкцию как их воссоздать. И путешествовать таким образом буквально со скоростью света. Причем совсем не энергозатратно. Я где-то читал, что строящийся радиотелескоп Square Kilometre Array будет способен посылать сигналы на 200 тысяч световых лет. То есть, на всю нашу галактику. И, соответственно, будет способен принять сигналы от такого же радиотелескопа. Так что в пределах сотен тысяч световых лет (в пределах одной галактики) относительно развитые цивилизации вполне способны общаться и строить союзы "лесник", предупреждая других о враждебных цивилизациях. Ну как общаться... Обмениваться банками данных или передавая местный вариант ИИ, если еще не способны оцифровать сознание.


  1. Tsimur_S
    01.08.2021 18:18
    +2

    Пример с муравьями совершенно некорректен, он никак не иллюстрирует гипотезу темного леса из книги. Просто в виду отсутствия у муравьев разума.

    Ваш друг Иван совершенно точно знает что

    а) ущерб от набега муравьев на кухню малозначим. да и сам набег конкретно этих муравьев маловероятен

    б) он за одну минуту может уничтожить муравейник

    ему нету нужды превентивно их уничтожать. Он может это сделать в любой момент когда это станет актуальным.

    Будь же эти муравьи за 1000 световых лет и обладай они разумом близким к людскому то Иван бы заволновался. Он знает на опыте человечества что за 1000 мир перешел от повозок к космическим полетам. И некую цивилизацию, с неизвестным и возможно абсолютно чуждым менталитетом, сейчас скрывает "туман войны" в тысячу лет.

    Наше Солнце — звезда второго поколения.

      Третьего.

     Множество звезд родились, сияли и взорвались задолго до его появления. 

    Если бы не было этих звезд то на Земле бы не было металлов. Жизнь в том виде каком она есть сейчас была бы невозможна.

    Сложившие на Земле условия, приведшие к появлению жизни, так же нельзя назвать уникальными.

    Они совершенно точно не уникальны, с учетом огромного количества звезд но достаточно редки.

    Сложив эти факторы, мы приходим к выводу, что человечество далеко не первая возникшая в космосе цивилизация. Жизнь и разум возникли задолго до нашего появления.

    Спекуляция. Тут ровно 50%: могла возникнуть а можем и мы быть первыми. Даже в бесконечно большой вселенной кто-то же должен быть первым? Каких-либо наблюдений в пользу первого или второго факта у нас нет.


    1. Zangasta Автор
      01.08.2021 18:35

      Тут ровно 50%: могла возникнуть а можем и мы быть первыми.

      Спасибо, посмеялся. Это отсылка на анекдот про динозавра на Невском?

      На самом деле шанс в 50 процентов будет только если во всей огромной, многомиллиардлетней вселенной будет РОВНО ДВЕ разумные цивилизации. В этом и только в этом случае первыми будем либо мы --- либо они.

      Во всех остальных случаях наши шансы ниже.


      1. Tsimur_S
        01.08.2021 18:45
        +1

        Спасибо, посмеялся. Это отсылка на анекдот про динозавра на Невском?

        Да, это он и есть.

        Во всех остальных случаях наши шансы ниже.

        А вот это уже совершенно точно не правда. Есть еще ситуация где мы единственная цивилизация, соответственно мы первые со 100% вероятностью.


        1. Zangasta Автор
          01.08.2021 18:47
          -2

          В этом случае понятие первая теряет смысл.


          1. perfect_genius
            01.08.2021 19:43

            Почему не может возникнуть вторая?


            1. Zangasta Автор
              01.08.2021 19:53

              Единственная станет первой именно в момент возникновения второй, не раньше.


              1. Osnovjansky
                02.08.2021 00:08
                +2

                Если я иду и собираю грибы, то первый гриб станет первым только после нахождения второго? У вас странная логика.


                1. Zangasta Автор
                  02.08.2021 10:38

                  У меня обычная логика. Вот что например, ты будешь делать с со своим первым грибом если второго НЕ БУДЕТ?


                  1. Osnovjansky
                    02.08.2021 12:44
                    +1

                    Вот что например, ты будешь делать с со своим первым грибом если второго НЕ БУДЕТ?
                    То же самое. Разве будущее влияет на настоящее? Первый гриб становится первым в момент обнаружения, а не после того как появляется второй.

                    Я, кажется понял, в чем разница взглядов. Для меня порядковые числительные и количественные — принципиально разные шкалы измерения. Для вас — почти одно и то же.
                    В результате, для вас фраза «первый и единственный» — тавтология, а для меня — уточнение


                    1. Zangasta Автор
                      02.08.2021 12:58
                      -5

                      Природа не любит тавтологии.

                      Высказывания: "Я нашел гриб" и "Я нашел первый гриб" несут одинаковое количество информации.

                      А бессмыслицу нужно сокращать.


                      1. Zangasta Автор
                        02.08.2021 15:41
                        -1

                        Проиллюстрирую для тех, кто не понял разницы.

                        1918 год. Сидящий в окопе солдат встречает путешественника во времени.

                        --- Надеюсь, мировая война станет последней в истории человечества, --- говорит солдат.

                        --- Ты про про первую мировую? --- уточняет путешественник.

                        Надеюсь я привел не слишком сложный пример.


                      1. Osnovjansky
                        03.08.2021 12:34

                        Если верить заметке в ЖЖ

                        Термин «Первая Мировая Война» (First World War) впервые был использован в 1933 году в названии фотоальбома «The First World War: A Photographic History» («Первая мировая война. Фотографическая история»), изданного под редакцией ветерана войны, американского журналиста Лоуренса Столлингса (1892-1968)
                        И, кстати, тут прямая аналогия с первой цивилизацией, с которой началась ветка. Слово «первый» подчёркивает, что «раньше такого не было»


                      1. Zangasta Автор
                        03.08.2021 12:59
                        +1

                        Слово «первый» подчёркивает, что «раньше такого не было»

                        Тут скорее подчеркивается, что это может повториться. Заведомо единичные события никто не называет первыми --- про распятие Иисуса никто не называет первым.


    1. gans2
      02.08.2021 10:45

      >Пример с муравьями совершенно некорректен,

      Именно потому, что муравьи в лесу, а не на кухне завелись.

      (или внутри ящика с коммутатором).


  1. emptycat
    01.08.2021 19:07

    Все эти размышления о вымышленном мире легко опровергает модель другого вымышленного мира - Warhammer 40000.


    1. T_Cirkla
      01.08.2021 21:38

      Обе модели маловероятны.


  1. NightShad0w
    01.08.2021 19:19

    Хороший разбор хорошей концепции.

    Однако, стоит помнить, что не всегда руководство в масштабах цивилизации осуществляется согласно здравому смыслу. Человеческий фактор никто не отменял, и пока Звездным Флотом не станет руководить идеальный мыслительный субъект, нацеленный на бесконечное развитие и существование, всегда остается шанс, что кто-то один(соразмерно внешней структуре управления и контроля, а не в одно физическое лицо), будет реализовывать свои личные планы и стратегии, никак не связанные с общей стратегией выживания.

    Это же работает на всех масштабах - эпоха географических открытий, колониальные режимы, этносы и завоевания, локальные этнические конфликты, экологические проблемы, межпланетная экспансия, дележка гипотетических ресурсов и их освоение. Долговременные планы и цели не всегда реализуются, несмотря на публичные заявления.

    Обратную ситуацию хорошо иллюстрирует 2 и 3 том Хроник Дюны, где всю обитаемую Вселенную пришлось поставить на колени ради выживания цивилизации.

    Психологическая игра "Красное-Черное", обогащенная трудностью коммуникации, и отсутствием второго шанса. Все знают, что надо ставить на черное, но идти на этот риск мало кто способен.

    А введение лесника не соответствует бритве Оккама.


    1. r4nd0m
      02.08.2021 11:26

      идеальный мыслительный субъект, нацеленный на бесконечное развитие и существование

      Разве такой субъект не закономерный результат эволюции? По-моему, логично предположить, что любая жизнь, не орентированная на межпланетную и межзвёздную экспансию, будет уничтожена своей звездой в относительно короткий по космическим масштабам промежуток времени.


      1. kogemrka
        02.08.2021 13:32

        Разве такой субъект не закономерный результат эволюции?

        А с чего бы эволюции оптимизировать в "бесконечное существование" или "развитие"?

        Эволюция - это оптимизация по выживаемости гена (тут можно поставить несколько звёздочек на это слово).

        Выживаемость гена в некоторых условиях может быть увеличена через "пусть популяция чаще обновляется (быстрее рождаются новые поколения и вымирают старые)", что строго противоположно "бесконечно существовать".

        Или же через "пусть в мозг меньше ресурсов идёт, а в сильные плавники - больше" - что резко обрубает возможность развития отдельного субьекта.

        По-моему, логично предположить, что любая жизнь, не орентированная на межпланетную и межзвёздную экспансию, будет уничтожена своей звездой в относительно короткий по космическим масштабам промежуток времени.

        Ну да. А у вас что, эволюция богом направляется? Это простой механизм - есть особи, есть ограниченные ресурсы, особи из пула, которые справляются хуже, умирают.

        У этого простого механизма нет заглядывания в будущее и какой-нибудь понималки возможности вспышки ближайшей сверхновой.


        1. r4nd0m
          02.08.2021 13:48

          А чем ваше "бесконечное существование" отличается от вашей же "выживаемости"?


          "пусть популяция чаще обновляется (быстрее рождаются новые поколения и вымирают старые)", что строго противоположно "бесконечно существовать".

          Для поколений — да, но не для самой популяции и генов.


          А у вас что, эволюция богом направляется?

          Нет, где вы такое у меня увидели?


          У этого простого механизма нет заглядывания в будущее и какой-нибудь понималки возможности вспышки ближайшей сверхновой.

          Да, вообще никакой понималки нет. Просто вид, колонизировавший космос, при планетарной катастрофе выживет, а остальные — не очень (хуже справились).


          1. kogemrka
            02.08.2021 14:35

            А чем ваше "бесконечное существование" отличается от вашей же "выживаемости"?

            Вы говорили "идеальный мыслительный субъект, нацеленный на бесконечное развитие и существование"

            Я говорю о выживаемости гена(тут много звёздочек к слову). Ген - это не субъект, и мыслить он не умеет. Целеполагания ему, также, не завезли.

            Да, вообще никакой понималки нет. Просто вид, колонизировавший космос, при планетарной катастрофе выживет, а остальные — не очень (хуже справились).

            Ну да, это разумные рассуждения.

            Но нет ни одной причины полагать, что вид, колонизировавший космос будет состоять из "идеальных мыслительных субъектов" или что эволюционный отбор вообще к ним приведёт.


            1. r4nd0m
              02.08.2021 15:05

              Вы говорили "идеальный мыслительный субъект, нацеленный на бесконечное развитие и существование"

              Я цитировал, и не утверждал, а спрашивал. Но не суть.


              Я говорю о выживаемости гена(тут много звёздочек к слову). Ген — это не субъект, и мыслить он не умеет. Целеполагания ему, также, не завезли.

              Ген не может мыслить, но не может и существовать без носителя, который может уметь мыслить. Мышление это колоссальное эволюционное преимущество, оно позволяет адаптироваться практически мгновенно. Я почти уверен, что на определённом этапе оно полностью заменит эволюцию генов. Но, я думаю, что уже сейчас очевидно, что только мыслящие и желающие развиваться существа могут выйти в космос (и уже там деградировать в ксеноморфов, шутка).
              Из этого можно сделать вывод, что цивилизации, выжившие в космических масштабах времени и враждебности окружающей среды, должны уметь очень хорошо мыслить.


  1. tbl
    01.08.2021 19:36
    +1

    Звёздные системы первого поколения не могли иметь цивилизации на орбите. В их звездах только начинался нуклеосинтез, никаких металлов не было. Звезды второго поколения только-только начинали генерировать элементы вплоть до железа. Солнце, кстати, это звезда третьего поколения. Поэтому, цивилизации могли появляться только после разрушения звезд второго поколения


  1. SebastianP
    01.08.2021 19:54
    +2

    Наше Солнце — звезда ТРЕТЬЕГО поколения. Углерод в наших телах выхлоп от четвёртого


    1. Zangasta Автор
      01.08.2021 19:55

      Спасибо, поправлю.


  1. axe_chita
    01.08.2021 20:15
    +1

    Интересная концепция, чем то похожа на вселенную «Звезды холодные игрушки», только лесники (Старшие расы) там не сильно мирные и добрые.


    1. Zangasta Автор
      01.08.2021 22:32
      +2

      Спойлер по сюжету!

      Настоящий лесник там Звездная тень


  1. thewall
    01.08.2021 20:43
    +2

    развитые космические цивилизации — они как борщевик. В смысле, их очень сложно уничтожить под корень. Всегда остаются отдельные ростки: форпосты, исследовательские группы, колонии…

    Вы путаете уничтожение вида/рода и уничтожением цивилизации.
    Пришельцам(назовём их так) достаточно нанести один серьёзный удар чтобы уничтожить цивилизацию. Скажем лишить человечества электричества, или уничтожить компьютерные технологии, причём не важно как, это просто понятные и наглядные примеры. Наша цивилизация посыпется сама, потому что будут нарушены многие процессы и связи из которых она состоит. Человечество начнёт «передел собственности» вспомнив все «старые обиды» и быстро скатится в каменный/рабовладельческий/феодальный век и… получится что:
    Часть из них спрячется, стремясь сохранить свой род.

    И всё, род/вид будет сохранён, но на уровне домашних животных или тараканов/моли, не предоставляющих особого интереса/вреда пришельцам.

    одновременный старт диктует примерно равные технические возможности.

    Расскажите это индейцам американского материка, туземцам Океании, или даже африканским бушменам, которые топтали Африку пока в Европе лежал лёд толстым слоем.

    Появление прямого и явного злодея Гитлера привело к союзу двух непримиримых конкурентов: СССР и США

    Во первых США и СССР после победы грызлись пока не осталось только США.
    Во вторых при «появление прямого и явного злодея» Кортеса, все старания американских индейцев ни к чему не привели и их цивилизация(и) были сметены как крошки со стола.

    И теперь на счёт «лесника»:
    Современным лесником является США. Они диктуют всем «что такое хорошо и что такое плохо». За это их не любят и даже ненавидят, и все только и смотрят чтобы свергнуть их и при возможности занять их место. Причём самое смешное то, что при всей показной, лицемерной ненависти, простые люди со всего мира стремятся именно туда.


    1. Zangasta Автор
      01.08.2021 21:11
      -2

      Скажем лишить человечества электричества, или уничтожить компьютерные технологии

      Как я вижу продолжение этой темы:

      --- Здравствуйте thewall. Мы злобные пришельцы. Мы прочитали вашу реплику на Хабре и решили нанять вас консультантом для уничтожения человечества. Нам нравится ваш подход! Вот только мы не очень понимаем, как лишить человечества электричества? Электростанции взорвать? Так они новые построят...

      --- Вы должны уничтожить электричество --- как силу природы! Как физическое явление!

      --- Здорово! Спасибо! Как мы сами не догадались! А как нам это сделать?

      --- Это не ко мне. Я решаю только стратегические вопросы.


      1. rsashka
        01.08.2021 21:27
        +2

        Это смотря как запретить. Даже сейчас РКН старается наложить запрет на пользование отдельными технологиями и сервисами, и это у него получается все лучше и лучше.

        А теперь представьте высокоразвитая цивилизация, не ограниченная рамками человеческих законов и морали, начинает уничтожать любых источники ЭМ волн, включая часть прилегающей территории. Люди сразу на свечки перейдут и изобретателей сами будут гнобить, чтобы окружающим не досталось.


        1. Zangasta Автор
          01.08.2021 21:38
          -3

          И эффективность этих мер будет в точности как у РКН. Которые из кожи вон лезут --- а рейтинг Кремля как падал, так и падает. Они уже даже в публичные дискуссии даже не вступают --- только на силовиках держатся.

          Так и со злобными пришельцами. Отменят электричество --- вернутся газовые горелки, микрофиши, бумажные книги и газеты.

          А буквально за считанные годы вернутся и компьютеры с интернетом --- на оптоволоконных кабелях и фотонной логике.


          1. thewall
            01.08.2021 21:46
            +1

            А буквально за считанные годы вернутся и компьютеры с интернетом — на оптоволоконных кабелях и фотонной логике.
            Это если межгалактический РКН будет сидеть на попе ровно и наблюдать как человечки изобретают «фотонной логику» и строит инфраструктуру. Вы действительно верите в то что они будут настолько наивны?
            1. Пришельцы такого не позволят.
            2. Как я раньше написал человечество само начнёт делёжку собственности и будет само себя уничтожать.


            1. Zangasta Автор
              01.08.2021 21:54
              -1

              Я действительно верю в то, что предложенный вариант, а именно: лишить человечество электричества не будет реализован ни при каких обстоятельствах.

              В том числе и потому, что для полноценной его реализации нужна магия.


              1. thewall
                01.08.2021 22:02

                для полноценной его реализации нужна магия.

                Так когда-то говорили и про полёт и про телевизор и даже про унитаз с центральным водоснабжением и канализацией.


              1. Ion_Storm
                03.08.2021 13:59

                "Сталь разящая" как фантастический, но реализуемый вариант.


          1. thewall
            01.08.2021 21:59
            +4

            А буквально за считанные годы вернутся и компьютеры с интернетом — на оптоволоконных кабелях и фотонной логике.

            Позвольте поинтересоваться, как ваши «компьютеры с интернетом — на оптоволоконных кабелях и фотонной логике» могут появиться без обычных компьютеров и/или электричества? Если вся современная промышленность, построенная между прочем на обычных компьютерах, будет уничтожена, то фотонной логике взяться не от куда. Вы даже банального «оптоволоконного кабеля» с нужными характеристиками создать не сможете, я уже молчу о чём то на порядки более сложном.


            1. Zangasta Автор
              01.08.2021 22:18
              -5

              Поздравляю. Вы только что доказали, что компьютеров не бывает.

              Для того, чтоб создать микросхему компьютера нужен компьютер. А значит первой микросхеме компьютера просто неоткуда взяться!

              Шах и мат! (Демонически хохочет)


              1. thewall
                02.08.2021 00:27
                +1

                Вы что-нибудь слышали про электронные лампы, транзисторы? Почитайте, вам будет интересно, узнаете массу нового. :)


            1. Wizard_of_light
              02.08.2021 10:15
              +1

              Но возможны же и неэлектрические вычислительные устройства. Пневматическая, гидроструйная, механическая логика. Да и оптика уже на обычных линзах позволяет осуществлять интегрирование, Фурье-преобразования, свёртки. Электроника просто позволила всё то же самое это сделать меньше, легче и быстрее, и вычислители на иных принципах ушли в очень узкие ниши, но в принципе есть и другие пути, труднее и менее эффективнее, но в отсутствие альтернативы...


              1. thewall
                02.08.2021 19:49
                +2

                Абсолютно верно. Есть только одно важное НО… а именно то что всё это требует времени и ресурсов, а у вас над головой полчища пришельцев закидывают вас «бомбами», вся промышленность разрушена и уцелевшие соплеменники рыскают в поисках чего пожрать. Им не до преобразования Фурье. Разницу чувствуете?


                1. Ion_Storm
                  03.08.2021 14:06
                  +1

                  Тут сложно предсказать, как поступит человечество при наличии явного внешнего врага. Может последние крохи будет собирать и отдавать ученым, тащить знания в бункеры для будущих поколений. В горных разломах строить лаборатории и ковать вундерваффе. Или как в "алой чуме", за два-три поколения деградирует до родоплеменной системы, скатится в анархию, а злых пришельцев будет почитать как богов и мифологизировать их.


                  1. thewall
                    03.08.2021 20:07

                    Предположу с вероятностью близкой к 100% что основная масса пойдёт по второму пути и будет всячески мешать тем единицам кто попытается сохранить знания. Ведь они не почитают «богов» пришедших с неба и несут ересь о том чтобы самим стать богами. А значит «не надо раскачивать лодку», «накликают божий гнев на наши головы», «лишь бы не было войны», «мы за стабильность», «сжечь еретиков!» ну и дальше в том же духе.


      1. thewall
        01.08.2021 21:39
        +1

        Я привёл простой пример только для наглядности, чтобы показать насколько «нежна» наша цивилизация и как легко её разрушить.

        Вы должны уничтожить электричество — как силу природы! Как физическое явление!

        Отнюдь. Но высшая цивилизаций теоретически может… скажем раскидать по поверхности земли некие источники электромагнитных импульсов, которые периодически будут гробить всю электротехнику в округе. И всё, «кина не будет — электричество кончилось»(с).


        1. T_Cirkla
          01.08.2021 21:55

          Это не имеет смысла. Предположим, есть звёздная цивилизация, сама по себе не воинственная, но которая обнаружила некую дозвёздную цивилизацию, которая может выйти к звёздам и якобы стать угрозой, и которую было решено держать на определённом уровне, чтобы она не выбралась к звёздам. Но вот тут как раз нелогичность и вылезает. Наличие таких факторов всё равно приведёт к тому, что цивилизация успеет уничтожить себя на планетном уровне, в междоусобных войнах. Ежели эти факторы исчезают и всё идёт к тому же, что фактически у той звёздной цивилизации, то держать вторую цивилизацию на определённом уровне, а то и уничтожать, неэтично, да и смысла нет.


          1. thewall
            01.08.2021 22:18

            Это не имеет смысла. Предположим, есть звёздная цивилизация, сама по себе не воинственная

            Это если «цивилизация, сама по себе не воинственная». Но надеяться на такое наивно, а потому надо быть готовым к худшему. Наивные долго не живут. А значит смысл есть.


            1. T_Cirkla
              01.08.2021 23:39

              Нужно быть готовым к худшему на уровне своей планеты в рамках внутрипланетных отношений. Вот тут да, наивность была бы странной.

              Воинственность — значит, есть сильная власть. Значит, есть противоборствующие группировки. Значит, при определённом уровне вооружений остаётся вероятность самоуничтожения, довольно большая, ещё на планете, в своей системе максимум.
              Но это всё всё равно гипотетически. Так что остаётся более вероятная опасность на своей планете.


              1. thewall
                02.08.2021 00:30

                Вот мы уже почти выбрались за пределы планеты. Ещё сотня лет и человечество закрепится на других планетах. Ещё 2-3 сотни и эти планеты захотят самостоятельности и будут за неё воевать. А несколько сотен лет в масштабах вселенной — мизер.


                1. DesertFlow
                  02.08.2021 00:56

                  Ещё 2-3 сотни и эти планеты захотят самостоятельности и будут за неё воевать.

                  А зачем и ради какой цели им воевать? К тому времени всю работу будут делать роботы, а значит не будет борьбы за легкую жизнь. Правительств и обладателей власти, соответственно, тоже не будет. Зачем, какой смысл? Кем можно командовать, если у людей и так все есть, а на разделяемые ресурсы (вроде пашни) проще скинуться небольшой общиной. Всю работу ведь будут делать роботы, никого не нужно заставлять там работать. Получается, что воевать захотят только отдельные преступники. С которыми легко справится коллективная система безопасности. Вот она точно останется, какой-то аналог полиции (тоже, скорее всего, роботизированный).


                  1. VIPDC
                    02.08.2021 09:06

                    Если только будет кому колонизировать. Динамика численности населения в развитом обществе именно которое и будет иметь возможность колонизировать, явно не растущая. Заселим разными роботами планеты и будут они там развиваться


                  1. thewall
                    02.08.2021 19:58

                    А зачем и ради какой цели им воевать?

                    За свою независимость. Мало кому нравится что ими управляют какие-то неизвестные чуваки с другого конца света, которых никто не видел. Вся человеческая история забита войнами за делёж территорий.

                    Правительств и обладателей власти, соответственно, тоже не будет.

                    Ой не факт. Может роботы и будут, но без правительства никак. Сильные мира сего никогда не отдадут власть за здорово живёшь. А даже если отдадут, то на их место придут другие.


        1. Zangasta Автор
          01.08.2021 21:57

          Вот уж чего чего, а с источником лектромагнитных импульсов человечество справиться сумеет. Найдутся умники которые будут от этих импульсов технику запитывать.


          1. thewall
            01.08.2021 22:10

            Найдутся умники которые будут от этих импульсов технику запитывать.

            1. Какую технику? Она вся уже будет безнадёжно испорчена. И создать её не на чем, потому что та техника испорчена так же.
            2. Чтобы умники нашлись им нужно как минимум время, инструменты и сам источник электромагнитных импульсов. Да вот только ничего такого у них не будет, а к тому времени когда оно возможно появится(соберут с мира по нитке), то наша цивилизация уже будет уничтожена и не будет ресурсов чтобы выпустить противоядие в промышленных масштабах. А сидя в пещере сильно не посопротивляешься.


            1. Zangasta Автор
              01.08.2021 22:26

              Какую технику? Она вся уже будет безнадёжно испорчена. И создать её не на чем, потому что та техника испорчена так же.

              Мне это напоминает мой спор с одним современным писателем. Он мне доказывал, что если меня лишить современной техники и выбросить в лес --- то я там помру от скуки.

              --- А с чего это я помру? Возьму провода, стеклянные банки, всякий хлам и построю себе ламповый радиоприемник. Не ахти какое развлечение, но уже веселей...

              --- НЕТ! Никто не может сам сделать радиолампу!

              --- А откуда взялись по вашему первые радиолампы?


              1. tbl
                01.08.2021 22:33
                +2

                Чтобы сделать плохонькую лампу, нужен форвакуумный насос, если нужна хорошая, то впридачу еще и паромасляный.


                1. Zangasta Автор
                  01.08.2021 22:45
                  +1

                  На самом деле вполне хватает велосипедного насоса. Я говорю так уверенно, поскольку уже делал это когда ходил в кружок еще в СССР.

                  Лампа, конечно, выйдет недолговечная и некачественная. Но с поставленной целью справится.


                  1. iliasam
                    01.08.2021 23:47

                    А нить накала из чего предполагается делать?


                    1. Zangasta Автор
                      02.08.2021 00:00

                      Железо никель нихром.


                  1. agat000
                    02.08.2021 08:41

                    Лампы для перфекционистов. Детекторный приемник - выбор настоящего самоделкина ))) Делал в школе. Работал. Только динамик был заводской, из телефона.


              1. thewall
                02.08.2021 00:36
                +1

                мой спор с одним современным писателем. Он мне доказывал, что если меня лишить современной техники и выбросить в лес — то я там помру от скуки.
                Писатель ошибался только в одном — вы умрёте там не от скуки, а от голода.

                Ламповый радиоприемник он себе сделает, а-га. Первые несколько дней с пустым брюхом вам будет не до радиоприемника, а потом… потом вы умрёте.


                1. tbl
                  02.08.2021 10:36

                  да, даже если есть навыки добывания пропитания, то романтика закончится с первой зимой


                1. Zangasta Автор
                  02.08.2021 10:40

                  Первые несколько дней с пустым брюхом вам будет не до радиоприемника, а потом… потом вы умрёте.

                  В правительстве не дураки сидят. Я с собой еды возьму --- крупы, консервов, сухофруктов, пива. Условиями спора это не запрещено.


                  1. tbl
                    02.08.2021 16:08

                    если только этот комплект, то цинга обеспечена, если хвою жевать не будешь


                    1. Zangasta Автор
                      02.08.2021 16:10

                      Это, простите, вредное заблуждение.

                      В процессе консервации овощей и фруктов, в них сохраняется довольно много витаминов, особенно это касается витамина С. Конечно, значительная часть разрушается при обработке, но даже в этом случае конечные потери составляют не более 20-25%. Другие витамины, такие как бета каротин (витамин А), тиамин (витамин В1), рибофлавин (витамин В2) почти полностью сохраняются при мариновании.


                  1. thewall
                    02.08.2021 22:57

                    В правительстве не дураки сидят.

                    В каком именно?

                    Я с собой еды возьму — крупы, консервов, сухофруктов, пива. Условиями спора это не запрещено.

                    Тут речь о человечестве как о цивилизации. Ваша персона никому не интересна. Интересно только где вы возьмёте «крупы, консервов, сухофруктов, пива» на оставшиеся годы своего существования? Ведь «в правительстве не дураки сидят» и вам уж точно ничегошеньки не достанется, да ещё и конфискуют, если что заныкали.


                    1. Zangasta Автор
                      02.08.2021 23:08

                      Ваша персона никому не интересна. Интересно только где вы возьмёте «крупы, консервов, сухофруктов, пива» на оставшиеся годы своего существования?

                      У нас существует некоторый запас продуктов (зерна, картошки, крупы) которого хватит на год.

                      Соответственно за год нужно будет обеспечить его воспроизводство. Не вижу в этом ничего невозможного. Техника осталась, семена и скот тоже.


              1. unC0Rr
                02.08.2021 13:35

                А нынче есть вообще чего ловить в эфире? Радиостанции в мире вовсю на цифру переходят.


      1. Wizard_of_light
        02.08.2021 14:51

        Ну ладно, прямо сложно всё. Прикидываешься добрым, смотришь коэффициенты по социодинамическим таблицам и подкидываешь пару технологий с подковыркой, и такой "ну, вот инструкция, направлять туда, жать сюда, пока-пока, мне ещё десяток звёзд исследовать, развивайтесь тут, через сотню лет загляну". А через сто лет прилетаешь и "ах ты, целых полпланеты необитаемая? Удивительно что не вся. Ну как же вы так неосторожно, вот вам ещё пара технологий, с ними вы быстро всё исправите, я ещё лет через пятьдесят загляну посмотреть, вдруг уцелело чего".


    1. T_Cirkla
      01.08.2021 21:47

      Есть одно но. Воинственная цивилизация не выберется в космос в полном смысле этого слова. Она успеет себя уничтожить до планетарного объединения. То же самое касается и землян. Если невероятно предположить, что таки есть в качестве исключения воинственная звёздная цивилизация, которая вдруг, внезапно, захочет уничтожить цивилизацию землян, которая успела тоже стать почти звёздной, то это будет означать, что то, что позволило землянам измениться и построить почти звёздную цивилизацию, позволит после технокраха и восстановить технологическую составляющую цивилизации. Но а если этого нет, то и уничтожать нас никакого смысла, ибо это будет означать, что к этому моменту мы либо останется на уровне отдельных внутрипланетных государств на пути самоуничтожения, либо уже успеем сами себя уничтожить.

      Но всё это маловероятная гипотетическая ситуация.


      1. thewall
        01.08.2021 22:32

        Воинственная цивилизация не выберется в космос в полном смысле этого слова.

        С чего это вдруг? Любая цивилизация воинственная до определённого этапа. Мы например воинственная цивилизация. И мы на границе выхода. Человечество с древних времён режет глотки своим соседям, только перья летят. Официальная причина — они другие. Реальная причина — ресурсы.
        Возможен вариант, что при выходе в космос и доступе к безграничным ресурсам человечество наконец наладит распределение «каждому по потребностям», тогда оно успокоится на некоторое время и поменяет менталитет на более мирный, креативный, творческий. Но вот при встрече с пришельцами, снова возникнет борьба за ресурсы и причина «они другие» заиграет свежими красками. И вновь кровища и… что там в жилах у тех «рептилоидов» :)… польётся реками.


  1. T_Cirkla
    01.08.2021 21:22
    +3

    В целом океане положительных отзывов о Тёмном лесе как раз и не хватает критики.


    1. Tsimur_S
      01.08.2021 23:09
      +2

      Если это не сарказм то критики самой книги просто вагоны можно нагружать. А уж что касается последней книги то там на баттхертной тяге можно запускать флот для перехвата трисоляриан.


      1. agat000
        02.08.2021 08:53

        Значит хорошая книга. Если вызывает массовые дискуссии о применимости главной идеи.

        А то бывает прочитаешь хорошую интересную книгу и через месяц не можешь вспомнить о чем она.


        1. maksimtnt
          02.08.2021 10:40

          Да обычная фантастика, просто разрекламировали хорошо на мировом уровне. Читать скучно, персонажи никакие. Разве что темы интересные подняты и масштабы повествования большие.


        1. Tsimur_S
          02.08.2021 10:48

          Если вызывает массовые дискуссии о применимости главной идеи.

          Ну основные дискуссии вызывает как раз таки не главная идея книги. С ней то как раз все хорошо.


          1. agat000
            02.08.2021 11:51

            А здесь мы что обсуждаем? Разве не "теорию темного леса"?

            С ней как раз все плохо. Пессимистическая она.


            1. Tsimur_S
              02.08.2021 12:29

              А здесь мы что обсуждаем?

              По большей части здесь мы натягиваем сову на галактический глобус.

              С ней как раз все плохо. Пессимистическая она.

              Пессимистичность означает что теория плохая?


              1. Zangasta Автор
                02.08.2021 13:05
                +1

                Пессимистичность означает что теория плохая?

                На самом деле да.

                Эта тема для отдельной статьи, но если предельно упростить, то пессимистичность --- признак плохой теории. Это показатель, что принимающие её люди попали в ловушку "Умной Эльзы", которая рыдала от того, что её будущий сын вырастет, пойдет за пивом в погреб, споткнется и сломает шею.

                У многих теорий нет никаких достоинств, кроме пессимизма. Они плохо описывают происходящее, недостоверны... а обсуждаем мы их только и исключительно потому, что они пессимистичны.

                Они вызывают у нас эмоциональный отклик и понеслась...


                1. Tsimur_S
                  02.08.2021 14:00

                  пессимистичность --- признак плохой теории

                  Начала термодинамики утверждают что мы замерзнем всей вселенной спустя n миллиардов лет. Теория относительности утверждает(ну по факту она не утверждает а принимает это за аксиому-предпосылку но тем не менее) что мы не можем коммуницировать и перемещаться быстрее скорости света, что так же не радужная перспектива с учетом космических расстояний. Чем не пессимизм?

                  Это все плохие теории?

                   У многих теорий нет никаких достоинств, кроме пессимизма. Они плохо описывают происходящее, недостоверны... а обсуждаем мы их только и исключительно потому, что они пессимистичны.

                  У теории Темного леса ровно столько же достоинств сколько у любых других теорий разрешения парадокса Ферми/Дрейка, за неимением абсолютно никакой информации. Ваша теория лесника его никак не пытается решить, она не самодостаточна.


                  1. Zangasta Автор
                    02.08.2021 14:09

                    Это все плохие теории?

                    Это уже спор ради спора начался. Из утверждения "пессимизм - признак плохой теории" не следует, что все пессимистические теории ложны.

                    И да --- не все теории разрешения парадокса Ферми/Дрейка одинаково достоверны. Просто потому что вы не правы, утверждая что у нас нет "абсолютно никакой информации" --- некоторая информация у нас есть.

                    Мы наблюдаем нас.


                    1. Tsimur_S
                      02.08.2021 14:34
                      -1

                      Это уже спор ради спора начался. Из утверждения "пессимизм - признак плохой теории" не следует, что все пессимистические теории ложны.

                      Утверждение должно доказываться, хотя бы на примерах. Так или иначе все фундаментальные физические законы в чем то ограничивают безудержный полет фантазии. А значит они все пессимистичны. Теории вида: термояд в стакане воды, струнный транспорт, вечный двигатель наоборот глубоко оптимистичны.

                      Как минимум повод пересмотреть пессимизм как критерий плохости теории.

                       Просто потому что вы не правы, утверждая что у нас нет "абсолютно никакой информации" --- некоторая информация у нас есть.

                      Мы наблюдаем нас.

                      Эта информация как-то помогает выбрать решение парадокса Ферми, кроме как пальцем в небо?


                      1. Zangasta Автор
                        02.08.2021 14:38
                        -1

                        Эта информация как-то помогает выбрать решение парадокса Ферми, кроме как пальцем в небо?

                        Да.

                        Гуглите Байесовский анализ .


                      1. Tsimur_S
                        02.08.2021 15:01

                        Байесовский вывод — статистический вывод, в котором свидетельство и/или наблюдение используются, чтобы обновить или вновь вывести вероятность того, что гипотеза может быть верной

                        Может у вас завалялись наблюдения или свидетельства в пользу одной из гипотез?


                      1. Zangasta Автор
                        02.08.2021 15:30

                        Попробуйте оценить "по Байесу" два высказывания:

                        Наша цивилизация уникальна в рамках вселенной.

                        Наша цивилизация типична для вселенной.

                        Каждое из эти высказываний может оказаться верным. Но, очевидно, с разной вероятностью.

                        Пока понятно?


                      1. Tsimur_S
                        02.08.2021 15:44

                        Да. Что дальше?


                      1. Zangasta Автор
                        02.08.2021 15:52
                        -1

                        А это уже конец

                        Мы обсуждали высказывание:

                        У теории Темного леса ровно столько же достоинств сколько у любых других теорий разрешения парадокса Ферми/Дрейка, за неимением абсолютно никакой информации. (с)

                        Я доказал что не все теории разрешения парадокса Ферми/Дрейка одинаково достоверны.

                        Из этого следует вывод, что ваше высказывание не верно.


                      1. Tsimur_S
                        02.08.2021 16:06
                        +1

                        Я доказал что не все теории разрешения парадокса Ферми/Дрейка одинаково достоверны.

                        Так вы же не доказали ничего а приняли за очевидный факт.


                      1. Zangasta Автор
                        02.08.2021 16:14
                        -1

                        Потому что это очевидный факт. Высказывания:

                        Наша цивилизация уникальна в рамках вселенной.

                        Наша цивилизация типична для вселенной.

                        Имеют разную вероятность реализации.


                      1. Tsimur_S
                        02.08.2021 16:50
                        +1

                        Какие-нибудь обоснования этого очевидного факта будут?


                      1. Zangasta Автор
                        02.08.2021 17:21
                        -1

                        Нет.

                        Я не люблю подобных ответов, но обучать вас Байесовскому анализу не входит в мои планы.


                      1. Tsimur_S
                        02.08.2021 17:46

                        Байесовский анализ работает с изначальными вероятностями событий и фактами связанными с этими гипотезами. Пока что фактов от которых можно отталкиваться не приведено. Изначальные вероятности тоже отсутствуют.

                        Зато есть утверждение что вероятности не равны. Интересно что вы так подход "пальцем в небо" называете Байесом.


                      1. Zangasta Автор
                        02.08.2021 18:05

                        Пока что фактов от которых можно отталкиваться не приведено. Изначальные вероятности тоже отсутствуют.

                        Нет. Это вы их не видите.

                        Например то, что наше Солнце --- заурядная звезда третьего поколения --- вполне себе факт, пригодный для Байесовского анализа.


                      1. vedenin1980
                        02.08.2021 18:28

                        Потому что это очевидный факт. Высказывания:

                        1. Наша цивилизация уникальна в рамках вселенной.
                        2. Наша цивилизация типична для вселенной.

                        Имеют разную вероятность реализации.

                        Несомненно разную, но проблема в том, что имея одну нашу цивилизацию мы не можем знать какая вероятность больше. Чисто из теории вероятности.

                        Смотрите, мы знаем одного человека, который выиграл в лотерею миллиард долларов, и вообще ничего не знает об остальных участниках (вот даже не знаем участвовал ли кто-то еще и сколько вообще людей).

                        Можем ли мы заключить, что вероятность выиграть в лотерею миллиард долларов типична и доступна каждому?

                        наше Солнце — заурядная звезда третьего поколения — вполне себе факт

                        Тот факт, что победитель лотереи — вполне заурядный человек, обзначает ли что победить в лотерею — заурядное событие?

                        P.S. Вообще я видел разные выкладки о вероятности зарождении разумной жизни на какой-то планете. По некоторых, такое событие может иметь такую вероятность что даже один раз за время существования Вселенной на одной планете наблюдаемой Вселенной — большое везение.


                      1. Zangasta Автор
                        02.08.2021 19:50

                        Можем ли мы заключить, что вероятность выиграть в лотерею миллиард долларов типична и доступна каждому?

                        Нет не можем. Просто потому, что мы используем дополнительную информацию --- по условиям мы вообще ничего не знаем об остальных участниках. Зато мы знаем что такое лотерея и что такое миллиард долларов. Этого достаточно для анализа.

                        Еще раз --- не спорьте, а прочитайте что такое Байесовский анализ. Это отличный инструмент для таких вот подсчетов.


                      1. vedenin1980
                        02.08.2021 20:16
                        +1

                        Я знаю, что это, у меня диплом математика. Однако Байесовский вывод — метод статистики, а единственный случай существования разумной жизни НЕ МОЖЕТ быть ПРЕДМЕТОМ рассмотрения НИ одного СТАТИСТИЧЕСКОГО метода.

                        Зато мы знаем что такое лотерея и что такое миллиард долларов.

                        Как раз не знаем, мы инопланетяне нам слова лотерея и миллиард долларов ничего не говорят, у нас есть лишь единственный пример, который мы рассматриваем.

                        Более того, сам метод рассматривание выбрал именно того кто выиграл миллиард долларов в лотерее (мы живы — поэтому мы выиграли в лотерее), однако у нас нет реальных данных как часто выигрывают в данной лотерее и вообще есть ли в ней другие участники.

                        Условно вы пытаетесь построить прогнозы, по ОДНОМУ единственному случаю, причем у вас нет возможности провести ни одного подтверждающий эксперимент. Это равносильно киданию монетки.

                        Вот например, Гипотеза уникальной Земли, где вполне логично приводятся доводы к тому что разумная жизнь крайне редкое явление.


                      1. Zangasta Автор
                        02.08.2021 22:40

                        Условно вы пытаетесь построить прогнозы, по ОДНОМУ единственному случаю, причем у вас нет возможности провести ни одного подтверждающий эксперимент

                        Все у нас есть.

                        К возникновению жизни на Земле привели несколько факторов --- тип звезды, орбита планеты и прочее.. Проанализировав их мы приходим к выводу, что ничего уникального в них нет.

                        Значит подобных Земле миров где могла бы возникнуть жизнь множество.

                        Следующий вопрос --- Как часто возникает жизнь? Это тоже можно проверить экспериментальным путем. И это тоже было сделано в эксперимент Миллера — Юри.

                        Эти факторы не являются решающими --- но они существенно меняют вероятности рассматриваемых гипотез.


                      1. vedenin1980
                        02.08.2021 23:51

                        К возникновению жизни на Земле привели несколько факторов — тип звезды, орбита планеты и прочее… Проанализировав их мы приходим к выводу, что ничего уникального в них нет.

                        В каждом по отдельности — нет. В их сочетании может дать очень небольшую вероятность.

                        Уравнение уникальной земли:
                        image
                        N * — количество звезд в Млечном Пути (100 — 500 млд)
                        ne- среднее количество планет в обитаемой зоне звезды,
                        fg — доля звезд в галактической обитаемой зоне
                        fp — это доля звезд в Млечном Пути с планетами.
                        fpm это доля планет, которые являются каменистыми («металлическими»), а не газообразными.
                        fе — это доля обитаемых планет, на которых возникает микробная жизнь.
                        fc это доля планет, на которых развивается сложная жизнь.
                        fl — это часть общей продолжительности жизни планеты, на которой присутствует сложная жизнь.
                        fm — это часть обитаемых планет с большой луной. Если теория гигантского удара о происхождении Луны верна, эта доля мала.
                        fj — это часть планетных систем с большими планетами-гигантами.
                        fme- доля планет с достаточно низким числом вымираний.

                        Вот по этому уравнению — кол-во планет с сложной жизнь (даже не разумной) в нашей Галактике может быть от 0.1 — 1 (от одна планета в Галактике до одной планеты на 10 Галактик).

                        Это, конечно, не уникальность в рамках Вселенной, но все равно при ограничении перемещений скоростью света две цивилизации в разных Галактиках вымрут скорее всего намного раньше, чем успеют хоть как-то пообщаться, не то чтобы встретиться.

                        Значит подобных Земле миров где могла бы возникнуть жизнь множество.

                        В наблюдаемой Вселенной — возможно, в нашей Галактике — совсем не факт.

                        Эти факторы не являются решающими — но они существенно меняют вероятности рассматриваемых гипотез.

                        Угу, только вероятности меняются от одна планета с разумной жизнью на миллиард Галактик до планеты с разумной жизнью встречаются по несколько штук в рамках одной Солнечной системе (например, в нашей Солнечной системе жизнь может существовать в верхних слоях Венеры или в подледном океане Титана).

                        С таким разбросом любые оценки сводятся к Вере, а не научному методу.
                        Хочется, чтобы инопланетные цивилизации были за каждым углом и он подбирает вероятности такими, что они станут близкими к 1 для каждой Солнечной системы, не хочется — не сложно подобрать вероятности, чтобы наша планета была единственной в рамках нашей Локальной группы Галактик, к примеру.


                      1. Tsimur_S
                        02.08.2021 21:13
                        +1

                        заурядная звезда третьего поколения --- вполне себе факт, пригодный для Байесовского анализа.

                        Ну не такая уж и заурядная. Звезд типа G не более 5%. Если отобрать из них звезды только одиночные звезды третьего поколения то наверное до 1% упадет. Теперь нужно выбросить все звезды находящиеся в центре галактике(постоянные гамма-всплески будут часто убивать жизнь) и совсем уж на окраине. Итого количество таких звезд будет сильно меньше 1%. Как видно из https://ru.wikipedia.org/wiki/Аналоги_Солнца их не то что бы много в пределах 50 св лет от нас, около 20 штук .

                        Ну и как этот факт изменил в количественном виде вероятность гипотезы что мы уникальны?


              1. agat000
                03.08.2021 05:50

                По большей части здесь мы натягиваем сову на галактический глобус.

                Это да. Сова галактическая, поэтому натягивать интересно.

                Пессимистичность означает что теория плохая?

                Я не говорил, что теория плохая, я сказал, что с ней все плохо, это был ответ на

                С ней то как раз все хорошо.

                Теория спорная, как раз потому, что пессимистическая. Не любят люди, когда их не любят. Мы рвемся в космос к братьям по разуму (или к нормальным соперникам), а не к охотникам и лесникам.


  1. Polarisru
    01.08.2021 23:32

    У звезд первого поколения вряд ли могла возникнуть жизнь по причине крайней скудности химического состава ранней Вселенной.


    1. Zangasta Автор
      01.08.2021 23:43

      Да, у меня ошибка.
      Я пропустил первое звездное поколение. Сказанное, соответственно, относится ко второму и третьему звездным поколениям.


  1. thecove
    02.08.2021 09:08
    +1

    Ни одна из этих звездных империй не имела подавляющего превосходства — одновременный старт диктует примерно равные технические возможности.

    "одновременно" это по меркам вселенной +/- миллион лет. В то время как разница в 100 лет технологического развития уже дает абсолютное преимущество.


    1. Zangasta Автор
      02.08.2021 10:49
      +1

      Это если предположить, что технический прогресс --- величина постоянная.

      А это не так. Мы уже наблюдаем замедление после аномального периода бурного роста. Наивные прогнозы 60тых о водопаде открытий не оправдались. Космическая одиссея 2000 года не состоялась и Лунной базы у нас нету.

      Каждый шаг дается нам тяжелее и медленнее. Очень скоро мы выйдем на плато.

      Таким образом цивилизации возрастом в 100 тысяч и миллион лет отличаются крайне незначительно.


      1. XeaX
        02.08.2021 14:40

        Таким образом цивилизации возрастом в 100 тысяч и миллион лет отличаются крайне незначительно.

        Очень смелое утверждение, впрочем как и это:

        разница в 100 лет технологического развития уже дает абсолютное преимущество.


        1. thecove
          03.08.2021 06:24
          +1

          поскольку другого источника информации кроме человеческой истории у нас нет, то все постулаты и тезисы мы подкрепляем на основе анализа именно нашей истории.

          Осмелюсь утверждать что технологическое развитие ушедшее на 100 лет вперед дает неоспаримое преимущество перед противником.

          Тут конечно можно подискутировать, скатившись в глубокую древность, когда тысячи лет рубили друг друга палками и каменными топорами. Или в античность когда менее развитые но более многочисленные племена "выносили" империи. Но в целом ( если не натягивать сову на глобус ) то сто лет технологического развития это много.

          500 лет очень много, 1000 - это просто гигантская пропасть.

          А что такое для Вселенной 100, 500, 1000 лет?


      1. Tsimur_S
        02.08.2021 14:47
        +1

        Это если предположить, что технический прогресс --- величина постоянная.

        Технологический прогресс величина явно не постоянная. Но из этого никак не вытекает отсутствие разрыва между цивилизациями со 100 лет разницы.

        Очень скоро мы выйдем на плато.

        Мы уже выходили на плато и не раз. В 5 веке с окончательным развалом античного мира мы попали в плато, практически на тысячу лет. Даже не плато а скорее жесткий упадок которому очевидно предшествовало плато. Вплоть до эпохи Великих Географических Открытий. Там тоже долгое время все было не непрерывным ручьем а скорее наскоками, непрерывное шествие прогресса во всех отраслях началось только к концу 19 века. Сейчас появление крупных открытий замедлилось, хотя если сравнивать современные знания о космосе и знания 50х годов то это небо и земля.

        Почему бы плато не превратится в стремительный подъем лет через 100 если такое уже встречалось в истории?

        Ну просто навскидку - изобретут ИИ, более эффективный в науке и проектировании чем человеческий мозг. И заставят его проектировать следующее поколение ИИ.


        1. Zangasta Автор
          02.08.2021 14:58
          -2

          Почему бы плато не превратится в стремительный подъем лет через 100 если такое уже встречалось в истории?

          Потому что объём научных концепций конечен. Вне зависимости от того, будет или нет подъем через 100 лет, рано или поздно мы достигнем потолка.

          На этом революции в технике кончатся.


          1. Tsimur_S
            02.08.2021 15:51
            +2

            Потому что объём научных концепций конечен.

            Вы очень вольно подходите к исходным постулатам. Для меня максимум очевидно что это счетное множество. Ну допустим даже что оно конечно.

            Как из этого можно заключить что мы достигнем потолка? Это конечное множество может быть сильно больше того множества что мы постигнем за количество лет существования нашей цивилизации или нашей вселенной.


          1. thecove
            03.08.2021 06:18
            +1

            Потому что объём научных концепций конечен.

            Гёдель с вами не согласен


            1. Zangasta Автор
              03.08.2021 09:32
              -4

              Дорогой Гёдель.

              В нашей вселенной ограниченное число атомов. Она ограничена во времени.

              Следовательно, количество правил (физических законов) описывающих поведение этих атомов, так же конечно. Это число было бы конечно, даже если бы каждый отдельный атом жил по своим собственным физическим законам. Что далеко не так.

              Поскольку скорость научного познания растет, то любая долгоживущая цивилизация рано или поздно узнает все законы природы. После этого скорость познания падает до нуля.


              1. vedenin1980
                03.08.2021 10:07
                +1

                В известной нам сейчас наблюдаемой Вселенной.

                МетаВселенная может быть бесконечной и по времени и по пространству и содержать бесконечные наборы Вселеннных с своими физическими законами (тоже бесконечное количество).

                Кроме того для познания ВСЕГО нужно изучить все варианты биологической эволюции на каждой планете Вселенной (, очевидно все варианты жизенных форм и разумных цивилизаций не создать лишь вычислениями), что не Бесконечность, но очень большое число, особенно если путешествовать только со скоростью света.

                А значительная часть Вселенной уже за горизонтом событий, то есть пока нет сверхсветовых путешествий — они вообще недоступны уже, что делает познание всего невозможной.


              1. Tsimur_S
                03.08.2021 10:27
                +1

                Следовательно, количество правил (физических законов) описывающих поведение этих атомов, так же конечно. 

                Возможно ли описать движение хотя бы одного электрона конечным числом правил? С точными координатами и значением импульса одновременно.

                Поскольку скорость научного познания растет

                Каждый шаг дается нам тяжелее и медленнее. Очень скоро мы выйдем на плато.

                Тут никакого противоречия нету?


                1. Zangasta Автор
                  03.08.2021 11:57

                  Тут никакого противоречия нету?

                  Никакого противоречия тут, естественно, нет.

                  Технический прогресс и рост научных знаний --- разные явления.

                  Рассмотрим технический прогресс на примере самолета. Сразу после создания рост технических характеристик был экспоненциальный. Первые самолеты неуверенно взлетали и не имели никакого практического применения. Вот только они очень быстро стали верткими и прочными. На них стали сражаться, возить грузы и пассажиров.

                  На этом развитие замедлилось. Самолеты достигли плато где-то в 50-60. Все остальное --- не революционные прорывы, а небольшие улучшения.

                  Тоже самое и с компьютерами. Когда я впервые перешел от бумажных ведомостей в эксель --- это был прорыв. Я увеличил свою производительность в десяток раз. При этом эксель был практически тот же самый -- 30 лет развития не добавили ему практически ничего.

                  Поэтому я уверен, что технологическое развитие скоро остановится. Мы выйдем на плато по всем доступным нам технологиям.


                  1. Tsimur_S
                    03.08.2021 18:12

                     Поэтому я уверен, что технологическое развитие скоро остановится. Мы выйдем на плато по всем доступным нам технологиям.

                    Эта позиция предельно понятна.

                    Рассмотрим технический прогресс на примере самолета.

                    А вот эта нет. Что в примере с самолетом является скоростью научного познания? Разве это не то же самое что и тех прогресс? Разве в вашем же примере прогресс не является первой производной от объема знаний? Тех прогресс замедляется для меня значит что замедляется прирост объема знаний. Вторая производная от научного познания или ускорение - меньше нуля.

                    Иными словами: я так понял что объем знаний растет, а прирост этих знаний уменьшается, но что за скорость научного познания, которая так же растет, я не понимаю.


                    1. Zangasta Автор
                      03.08.2021 19:54

                      А вот эта нет. Что в примере с самолетом является скоростью научного познания?

                      Ничего.

                      Самолет иллюстрирует прогресс.

                      Научное познание --- это набор значимых фактов о мире. Скорость научного познания --- показатель, сколько этих фактов мы узнаем за единицу времени.

                      Очень важным фактором тут является открытие физических законов. Пример --- Ньютон открыл закон тяготения и всё. Нам больше не нужно замерять скорость падения всех вещей во всех мирах --- мы можем её рассчитать.

                      Узнав все управляющие миром законы --- создав теорию всего --- мы можем закрыть науку за ненадобностью --- практический пользы она больше не принесет.


                      1. Tsimur_S
                        03.08.2021 19:59

                         Скорость научного познания --- показатель, сколько этих фактов мы узнаем за единицу времени.

                        Тогда она должна падать, согласно вашей же теории, а не расти.


                      1. Zangasta Автор
                        03.08.2021 21:42

                        Тогда она должна падать, согласно вашей же теории, а не расти.

                        Чем более универсальны


                      1. Zangasta Автор
                        03.08.2021 21:50

                        Тогда она должна падать, согласно вашей же теории, а не расти.

                        Нужно учитывать важность фактов.

                        Первобытный человек узнал что камень падает.

                        Это один факт.

                        Ньютон вывел закон, которому подчиняются все предметы.

                        Это несоизмеримо большее количество камней и фактов.

                        Чем более универсальный закон мы открываем, тем больше объясняем себе картину мира. Тем больше фактов (единиц знания) мы получаем.


              1. thecove
                03.08.2021 12:07
                +1

                Поскольку скорость научного познания растет, то любая долгоживущая цивилизация рано или поздно узнает все законы природы. После этого скорость познания падает до нуля

                когда найтете все десятичные знаки числа π тогда и поговорим.


          1. kogemrka
            03.08.2021 18:05

            Потому что объём научных концепций конечен. Вне зависимости от того, будет или нет подъем через 100 лет, рано или поздно мы достигнем потолка.

            На этом революции в технике кончатся.

            Ну да.

            Допустим, объём научных концепций конечен. И количество возможных резких взлётов в прогрессе - тоже конечно.

            Хорошо. Представим нас и цивилизацию, у которой было два миллиарда сто пятьдесят миллионов пятьсот восемьдесят пять тысяч четыреста два взлёта, каждый из которых был значительнее и революционнее, чем тот, что мы наблюдали у себя за последний век. И на этом революция в технике у них закончилась.

            Конечное число? Конечное. Революции закончились? Закончились. Вроде бы, предпосылки ровно те, в которых вы убеждаете.

            Не понимаю, с чего бы предполагать, что мы будем не сильно отличаться от этой цивилизации? Кажется, два миллиарда сто пятьдесят миллионов пятьсот восемьдесят пять тысяч четыреста два - это довольно большое, пусть и конечное число.


      1. thecove
        03.08.2021 06:34

        Каждый шаг дается нам тяжелее и медленнее. Очень скоро мы выйдем на плато.

        кто вам это сказал? Вы не видите взрывного роста компьютерных технологий и в биологии последние 50 лет?

        Мы уже наблюдаем замедление после аномального периода бурного роста.

        разные направления развиваются с разной скоростью. Где то застой ( например термояд и ракетные технологии) а где то прёт. Но какое то новое открытие стрельнет так что и термояд попрет и ракетные технологии с запасом на 200-300-500 лет. Так уже было в нешей истории. Пружина сжимается чтобы дать новый толчок. Но когда она сжимается не означает застой во всём.

        Таким образом цивилизации возрастом в 100 тысяч и миллион лет отличаются крайне незначительно.

        разница будет огромной. У вторых будет как минимум больше энергетических ресурсов. Ведь они имели фору в 900 тысяч лет. все равно что сравнивать неандертальца с запасом дров для костра на зиму и современную АЭС.


  1. XeaX
    02.08.2021 09:08

    Первый шаг к устройству такого союза — оповещение потенциальных жертв об охотнике. Этот принцип настолько всеобъемлющ, что присутствует даже у неразумных существ. Да, я о криках испуга или боли, издаваемых особями, схваченными хищником. Саму жертву они не спасают, но помогают выжить другим членам стаи, закрепляя, таким образом, этот паттерн через эволюционный отбор.

    Как вам удалось применить подходы эволюционного отбора к цивилизациям? Сколько поколений "скрещиваний" и "отмираний" цивилизаций может привести к закреплению этого паттерна, учитывая, что отправка сигнала "боли" достаточно ресурсоёмка, а в случае нападения ресурсов скорее всего будет не хватать на оборону?


    1. Zangasta Автор
      02.08.2021 10:43

      Сколько поколений "скрещиваний" и "отмираний" цивилизаций может привести к закреплению этого паттерна?

      Нисколько. У разумных существа существует разум --- и мы можем додуматься до готовой концепции, а не придти к ней методом проб и ошибок.


      1. XeaX
        02.08.2021 10:58

        Концепция и конкретная реализация - сильно разные вещи. Эволюционный отбор как раз позволил выработать достаточный уровень громкости и понятность другим крика боли. Как эффективно вопить на весь космос о боли - та еще задача. В указанном произведении, как максимум, раскрывались координаты звездной системы.


  1. Arxitektor
    02.08.2021 09:58

    Человечество с древних времён режет глотки своим соседям, только перья летят. Официальная причина — они другие. Реальная причина — ресурсы.

    На данном этапе развития человечество найдет причина уничтожать друг друга даже если каждому выдать по необитаемой планете. Даже с необходимой инфраструктурой.

    Думаю обитаемых планет на каждого человека на земле хватит даже в нашей галактике.

    Куда смогут переехать отдельные личности или народы/нации чтобы им никто не мешал.


    1. ClearAirTurbulence
      02.08.2021 10:22

      Не все захотят. Да и кто их туда повезет?


  1. gans2
    02.08.2021 10:15

    "Простой и понятный... "(тм) "взгляд эфемера на Дамбу Гувера"

    А Лю вообще пишет памфлеты политотные. Такая же "научная" фантастика, как "Записки о Кошачьем городе" Лао Ше. Лаоваям не понять.


  1. valemak
    02.08.2021 10:37

    "Тёмный лес" - это просто одна из теорий, очень популярная прежде всего потому, что преподнесена в эффектном виде.

    А вообще, пока что как сама теория, так и вся её критика - это спекулятивное бла-бла-бла. Нет достаточного количества объективных научных данных, которые бы более-менее подтверждали чью-то точку зрения по этому вопросу. Так что, на сегодняшний день всё это на уровне теологической дискуссии.


  1. Winnie13
    02.08.2021 11:05
    +2

    Думаю, что концепция «тёмного леса» особенно популярна в недемократических обществах именно по причине основной скрепы таких обществ: каждый сам за себя.

    А так в жизни кооперация и солидарность намного перевешивают по выгодам конкуренцию.
    Собственно, наша планета когда-то была тёмным лесом, однако в этом лесу из агрессивных племён-охотников возникли как-то более-менее цивилизованные сообщества, которые со временем пришли к принципу преобладания мирного сосуществования сначала внутри себя, а потом и вовне себя; и прогресс в этой сфере продолжается. Даже более того, именно мирное сосуществование в итоге даёт эволюционное преимущество через силу — такие сообщества просто сильнее физически.

    Так что, если космос и кишит охотниками «тёмного леса», то это (по историческим меркам) не навсегда. И скорее всего, это светлое будущее космоса уже наступило.

    Собственно, вот с этой мыслью автора я согласен полностью: «Если в темном лесу тихо — значит за порядком следит лесник.»


  1. freestyler8
    02.08.2021 11:34

    Ни одна из этих звездных империй не имела подавляющего превосходства — одновременный старт диктует примерно равные технические возможности.

    Вот это не совсем правильная предпосылка. Старт-то одновременно, но от зарождения звезды до появления разумной жизни проходит огромное количество времени. Погрешность даже в +/- 1% даст тысячи лет. А теперь представьте мысленно сражение современного военно-морского флота с флотом, например, древней Греции. Да даже флот не нужен, тупо пара современных сторожевых катеров и судно поддержки с боеприпасами и топливом вполне способны перемолоть весь "греческий" флот.


    1. Zangasta Автор
      02.08.2021 11:42

      1. freestyler8
        02.08.2021 11:57

        Сорри, пропустил. Интересное мнение, жаль - ни подтвердить, ни опровергнуть нельзя, только через несколько тысяч лет узнаем сбылось или нет :-)


        1. Zangasta Автор
          02.08.2021 12:06

          А по моему можем.

          Прогресс реально замедляется. Население перестает расти. Закон Мура был откорректирован в 75 году и требует новой коррекции. Все прогнозы 50тых и 60тых о вертикальном прогрессе оказались ошибочны.


          1. freestyler8
            02.08.2021 12:49

            А мне кажется что человечество на пороге открытия, которое приведет к резкому рывку. Что это будет - не рискну предполагать.

            Ну и есть мнение, что наша цивилизация поспешила с выходом в космос. Точнее не то, чтобы поспешила, а прыгнула выше головы и на этом фоне дальнейшее кажется замедлением. Если бы Гагарин полетел в 1991-м, то сейчас ощущения замедления не было бы.


            1. Zangasta Автор
              02.08.2021 13:09
              -1

              Если бы Гагарин полетел в 1991-м, то сейчас ощущения замедления не было

              Давайте вообще уберем Гагарина и космос за скобки.

              Вот как выглядело наше время из 40вых. Цикл рассказов "Я робот" Азимова. В нем в 90тые годы мы имеем разумных позитронных роботов.

              И где они?


              1. Tsimur_S
                02.08.2021 13:45
                +1

                Вот как выглядело наше время из 40вых. Цикл рассказов "Я робот" Азимова. В нем в 90тые годы мы имеем разумных позитронных роботов.

                Из Азимова вообще слабый предсказатель. Если честно то его книги про роботов имели скорее детективную ценность чем НФ. Он не особо заморачивался с объяснением технологий(что его вообще сподвигло рассматривать позитроны как ключевую вещь для мозгов робота, кроме модного звучания слова). В этом плане Артур Кларк гораздо строже.

                Спойлер

                Люди у Азимова летают через гиперпространство и пользуются роботами но для звонков используют телефонные автоматы, при том что беспроводной телеграф и рации уже существовали в реальной жизни. Видно как, по мере развития прогресса, в более поздних его книгах начинают появляться факсы, компьютеры. Причем роботы это нечто простое, появившееся в 90х годах а компьютеры это наивысшее достижение технологий времен космической империи. Забавно что его космическая империя так и не осилила производство этих роботов и они там были на уровне легенды на протяжении десятков тысяч лет.


              1. freestyler8
                02.08.2021 13:54

                Ну роботы и сейчас есть, нет разумных. Вполне возможно, что ИИ появится в ближайшие 10 лет, тогда погрешность прогноза не так и велика. Нейросети, которые наверняка станут фундаментом для ИИ, сейчас развиваются очень активно.


  1. Lexxnech
    02.08.2021 11:39
    +1

    Гипотеза темного леса неявно базируется на предположении, что шмалять межзвездным оружием уничтожающим целые цивилизации одним попаданием возможно, а вот колонизировать другие системы — никак, вообще. И создать системы «мертвой руки» — тоже. Именно абсолютно невозможно, даже в режиме «всей цивилизацией в едином порыве работаем на благо колонизаторского флота» а не просто «это экономически неоправданно», поскольку в гипотезе речь идет о явно осознаваемом выживании вида и жизни в бесконечном ожидании прилета подарка от обнаруживших цивилизацию соседей. В противном случае схема «все прячутся друг от друга но мы их чудом заметили и щас прихлопнем одним решительным ударом» не работает.
    А попытки спасти гипотезу введением сверхцивилизаций которые могут себе это позволить, или изменить подход на уничтожение малоразвитых цивилизаций в зародыше пока не расселились, рушит на корню все изначальное обоснование от теории игр, возвращаясь к классическому подходу «мы придумали пришельцев, которые хотят угнать наш скот, сжечь наши дома и похитить наших женщин».


  1. Tsimur_S
    02.08.2021 12:25

    Гипотеза темного леса неявно базируется на предположении, что шмалять межзвездным оружием уничтожающим целые цивилизации одним попаданием возможно, а вот колонизировать другие системы — никак, вообще.

    Тут говорится о гипотезе в отрыве от книжки? Потому что там колонизация была более чем активной.

    сюжетный спойлер

    Трисоляриане летели покорять другую звездную систему, выходцы с земли(экипаж одного из кораблей) основали поселение на других планетах. Главные герои потом встречаются с их потомками.

    Зачем для гипотезы темного леса нужен этот постулат про невозможность колонизации? Это ни на что не влияет. Гораздо более важным предположением тут является постулат о невозможности сверхсветовых коммуникаций.

    явно осознаваемом выживании вида и жизни в бесконечном ожидании прилета подарка от обнаруживших цивилизацию соседей

    Именно это в книге и происходит и именно в этом и была идея автора КМК, информация о координатах родной планеты является самой оберегаемой и на случай разглашения есть способ окукливания защищающий на 100% но имеющий свою цену.


    1. Lexxnech
      02.08.2021 15:46

      Возможность колонизации, особенно в сочетании с «темнотой» леса делает невозможной победу одним решительным ударом. Любой цивилизации тогда будет выгодно колонизировать максимальное число систем и выработать хотя бы какие-то схемы обмена информацией, с поправкой на задержки, хотя бы в формате потоков новостей. Но в такой ситуации обоснование «от теории игр» больше не работает — поскольку никто никого одним ударом уничтожить не может, стратегия «всегда бей первым» уже не является универсально выгодной и строить свою политику на ней совершенно не обязательно. Атака перестает решать свою главную задачу — устранение угрозы.
      После этого «темный лес» перестает быть гипотезой, и становится фантазией, когда от рассуждений о том, почему быть инициатором конфликта это единственная выигрышная стратегия, переходим к рассуждениям «зачем инопланетянам хотеть геноцида».


      1. Tsimur_S
        02.08.2021 17:54

        Возможность колонизации, особенно в сочетании с «темнотой» леса делает невозможной победу одним решительным ударом. 

        Нет нужды. Фактически каждую колонию можно считать самостоятельной цивилизацией. К тому же это не только выстрел вслепую в сторону сигнала а изучение и зачистка близлежащих звезд. На Земле и Альфе Ц знали что результатом сигнала будет смертный приговор для обеих систем.

         

        Атака перестает решать свою главную задачу — устранение угрозы.

        Из того что вы не можете одним махом убить всех тараканов на кухне не следует что их не нужно уничтожать вообще.


        1. Lexxnech
          02.08.2021 19:52

          Фактически каждую колонию можно считать самостоятельной цивилизацией. К тому же это не только выстрел вслепую в сторону сигнала а изучение и зачистка близлежащих звезд.

          Извините, но сообщить о нападении все же проще, чем его произвести. Межзвездная связь может быть трудной и медленной, но она точно не медленнее и не сложнее атаки. Поэтому как не считай системы отдельными цивилизациями, но вот о наличии агрессора и направлении атаки все они узнают. А зачистки «ближайших систем» будет мало. Напоминаю — речь идет о сеттинге, в котором все понимают, что поставлено на карту и тупить не будут. И уж тем более не будут рассчитывать, что оппоненты тупят. И если можно еще в начале расселения принять стандарты для оповещения, наметить пути разлета и придумать системы мертвой руки — нужно любую космическую цивилизацию рассматривать как уже это сделавшую.
          Из того что вы не можете одним махом убить всех тараканов на кухне не следует что их не нужно уничтожать вообще.

          Вот об этом я и писал под
          становится фантазией, когда от рассуждений о том, почему быть инициатором конфликта это единственная выигрышная стратегия, переходим к рассуждениям «зачем инопланетянам хотеть геноцида».

          Основа гипотезы темного леса — цепочки подозрений, при которых «если не ты, то тебя». А отнюдь не геноцид тараканов. Геноцид тараканов начинается тогда, когда обоснование со стороны теории игр работать перестает, но от вывода отказываться не хочется, и вместо рациональной причины атаковать ищутся поводы. Да еще и с использованием «эмоциональных аналогий» вроде борьбы с тараканами.
          Это вообще стандартная проблема, когда из неверных предпосылок делается вывод, а после обнаружения проблемы с предпосылками вместо отказа от вывода начинается поиск новых «подходящих» предпосылок.


          1. Tsimur_S
            02.08.2021 20:35

             И если можно еще в начале расселения принять стандарты для оповещения, наметить пути разлета и придумать системы мертвой руки — нужно любую космическую цивилизацию рассматривать как уже это сделавшую.

            Ну и что? Что это меняет? Увидел планету заселенную с уровнем развития ниже твоего - устроил экстерминатус. Увидел кого-то более прокачанного - затаился. Мертвая рука в книге использовалась как средство взаимного уничтожения трисоляриан, это просто чистая случайность и глупость трисоляриан что у Земли оказалось такое средство воздействия. Оффтоп: Кстати мне больше всего из книги любопытно откуда трисоляриане знали о применяемом другими цивилизациями оружии.

            Как мертвую руку можно использовать в качестве страховки от неизвестного тебе противника? Если противник тебе известен то стреляй сразу на поражение, если можешь. Или затаись. Или закрывайся белым флагом светоскоростного двигателя, если знаешь что "Вас обнаружили", если в рамках сеттинга книги оставаться.

            Да еще и с использованием «эмоциональных аналогий» вроде борьбы с тараканами.

            А фактически все к этому и сводится в рамках концепции. Либо ты задавишь как таракана либо ты сам прячешься как таракан. Какое-то массированное вторжение и прочие полумеры тут бессмысленны. Эмоциональной окраски тут не предполагается. Можешь задавить то дави. Даже если там останутся другие колонии то все равно, один живой таракан и один мертвый лучший исход для вас чем два живых. Это увеличивает шансы выживания твоей цивилизации.


            1. Lexxnech
              03.08.2021 10:59

              Так, во первых, я обсуждаю не трилогию Цысиня, а гипотезу, которая рассматривается как применимая к реальному миру. Во первых трилогию я не читал, а во вторых, я читал спойлеры, что важной движущей силой сюжета, в сеттинге где конфликт выводится из нерушимости законов физики и невозможности сверхсветовых коммуникаций, являются магические софоны, которые нарушают законы физики и обеспечивают сверхсветовые коммуникации, и которые созданы сверхцивилизациями, которые активно поддерживают всеобщий конфликт. С таким же успехом в сеттинг можно было ввести Богов Хаоса, которые хотят что бы кровь лилась и галактика пылала, и которые любую попытку дипломатии контрят тем, что из варпа вылезает демон и откусывает дипломатам головы.
              Поэтому я рассматриваю исключительно логику цепочек подозрений и следующую из нее единственную рациональную стратегию бить первым. И вот в случае возможности колонизации она не является единственно рациональной. Вот в этом месте:

              Можешь задавить то дави. Даже если там останутся другие колонии то все равно, один живой таракан и один мертвый лучший исход для вас чем два живых.

              В «темном лесу» если возможна колонизация, никто не может знать, может он задавить или нет. И оппонент тоже этого не знает. И уже не доказано, что начало конфликта вместо дипломатии увеличит шансы на выживание, потому что теперь у нас рискованная стратегия дипломатии противостоит не надежной стратегии первого удара, а не менее рискованной стратегии начала войны. То есть рискованность дипломатии перестает быть непреодолимым препятствием, поскольку отказ от дипломатии тоже является рискованным выбором, который тоже может привести к вымиранию.
              Как мертвую руку можно использовать в качестве страховки от неизвестного тебе противника?

              Вот как раз потому, что с мертвой рукой атака перестает «увеличивать шансы выживания твоей цивилизации.» Если возможны системы «мертвой руки» то нужно уже доказывать что риск быть атакованным при попытке начать дипломатию выше чем риск получить по лицу «мертвой рукой» в ответ на атаку без предупреждения. Уже мало указать на наличие риска, их уже необходимо сравнивать. А при нашем уровне знаний это уже будет спекуляциями и фантазиями. То есть реально никаких выводов из гипотезы темного леса сделать нельзя, это такое же тыканье пальцем в небо как и любая другая футурология дальнего прицела.


              1. Tsimur_S
                03.08.2021 13:38

                Так, во первых, я обсуждаю не трилогию Цысиня, а гипотезу, которая рассматривается как применимая к реальному миру. 

                Ну окей.

                конфликт выводится из нерушимости законов физики и невозможности сверхсветовых коммуникаций, являются магические софоны, которые нару

                Это один из капитальных проколов автора. Именно то что сначала постулируются цепочки недоверия из-за дальности световой коммуникации а потом вводятся софоны. Кстати говоря он не единственный.

                Сверхсветовой спойлер

                Автор описывал раннюю вселенную как вселенную с мгновенным распространением света и при этом она была уничтожена войной. Значит возможность коммуникаций со скоростью выше света никак не влияет на цепочки недоверия.

                Причем ему ничего не стоило дать этим софонам полную автономность. На сюжет это повлияло бы очень слабо.

                Спойлер

                Разве что пришлось бы придумывать другие ходы летящего флота в виде реакции на происходящие на земле событи

                которые созданы сверхцивилизациями, которые активно поддерживают всеобщий конфликт

                А вот тут мы вас и поправим.

                Спойлер
                • Они созданы не сверхцивилизациями а цивилизацией стоявшей лишь на ступеньку выше человечества.

                • Цивилизация не планировала(да и не смогла бы) играть роль охотника в темном лесу. У нее были свои цели - колонизация.

                • Наиболее эффективный способ уничтожения планет - подсветить координаты планеты для охотников, с безопасного для себя расстояния. С Землей этот фокус бы не прошел ввиду непосредственного соседства. А вот Земляне такой фокус провернули просто ради прикола(ради проверки теории).

                • Эта цивилизация прямым текстом говорила что если отправить экспедиционный флот прямо сейчас к Земле то к моменту прилета его встретят примерно одинаковые по уровню развития силы и флот разгромят.

                • Софоны нужны были не для поддержания градуса безумия в конфликтах галактики. А исключительно для сдерживания тех прогресса землян и разведки.

                • Остальные цивилизации по какой-то причине не использовали эти софоны.

                В «темном лесу» если возможна колонизация, никто не может знать, может он задавить или нет. И оппонент тоже этого не знает. 

                Ну с точностью в 100% конечно не знает. Но капитан британской канонерки мог довольно точно прикинуть сможет ли он задавить население островка с туземцами или есть шанс огрести. На уровне цивилизации это можно оценить гораздо более точно. Я исхожу из неявного предположения что у цивилизации с колониями уровень колоний +- соответствует уровню метрополии или немного слабее. То есть невозможна ситуация - на уничтоженной колонии мы увидели летающие на химических двигателях ракеты, а с метрополии нам в ответку прилетает околосветовой астероид. Либо не будет такого разрыва либо другим колониям все равно на уничтоженных. Второе предположение: для того что-бы вообще рассматривать возможность войны с первым ударом нужно иметь некий уровень технологий гораздо, гораздо выше текущего.

                Условно нужно иметь возможность прилететь кораблем в некую нейтральную звездную систему и с одного удара уничтожить как минимум планету а не делать это прямо из системы. Таким образом сам факт удара ничем не поможет в ответных мерах. Если же нет то выгоднее сидеть и не высовываться. Даже если цивилизация противника не может угрожать вам в ответ она всегда может сыграть роль наводчика для более развитых цивилизаций. Поэтому я и не вижу смысла в системах "мертвой руки". Она работает если враг известен.

                Ну если речь идет о двух цивилизациях которые узнали друг про друга и имеют уровень развития примерно на одной ступени то да, мертвая рука это хороший вариант. Но уверен если этот уровень низкий то там и так войны не будет.


                1. Lexxnech
                  03.08.2021 14:11

                  Я исхожу из неявного предположения что у цивилизации с колониями уровень колоний +- соответствует уровню метрополии или немного слабее.

                  Основная проблема в том, что исходите вы из неявного предположения, что цивилизации хотят воевать потому что хотят воевать. Не из-за цепочек подозрений и риска вымирания, а вот просто по злобе и воинственности. Ну и потому что могут.
                  Потому что в описанных условиях дипломатия уже не несет таких глобальных рисков, по крайней мере по сравнению с нападением, пускай даже и скрытым. А дальше получится ситуация, когда одним воевать уже не нужно, а другим — еще бесполезно.


                1. vikarti
                  03.08.2021 15:10

                  То есть невозможна ситуация — на уничтоженной колонии мы увидели летающие на химических двигателях ракеты, а с метрополии нам в ответку прилетает околосветовой астероид

                  В одном литературном произведении мне встретился чуть другой сценарий


                  А ведет экспансию, нашли Б, там уровень чуть чуть выше тех самых
                  химических ракет. Решили что планета самим нужна. Зачистили от хоть как то высокоразвитой жизни (включая даже домашних животных) биологическим оружием и десантом.
                  Через 20 лет в системе появились чужие сверхсветовые корабли. Б на самом деле не была самостоятельной, их еще раньше включила в свой состав В. А теперь В сильно зла на А и за уничтоженную Б и за убитых своих граждан. Теперь вопрос уже чей уровень технологий и путь развития — выше — А или В. В итоге А не уничтожили до конца но они очень серьезно пострадали.


                  Похожий сценарий из другого произведения.
                  А ведет экспансию но при этом не так уж просто делать сверхсветовые межзвездные перелеты, обычно это один раз в жизни и в анабиозе.
                  Боевые корабли есть но они автоматические.
                  Нашли планету населенную чужими, отправили автоматы которые должны были выжечь эту планету — мол конкуренты.
                  Только чудом атака сорвалось. А потом выяснилось что это таки не родная планета этих чужих. И прилетело бы.


  1. r4nd0m
    02.08.2021 12:29

    Переведем этот художественный образ в более строгую форму.

    У Цисиня есть более строгая форма, зачем устраивать испорченый телефон?


    Первая аксиома: выживание является основной потребностью цивилизации.
    Вторая аксиома: цивилизация непрерывно растет и расширяется, но объем вещества во Вселенной остается неизменным.
    И ещё: чтобы из этих двух аксиом вывести базовую модель космической социологии, вам потребуются две важные концепции: цепочки подозрений и технологический взрыв

    По-моему, единственный спорный момент здесь "цепочка подозрений". Остальное более-менее логично, для уровня фантастического произведения.


    1. Zangasta Автор
      02.08.2021 13:13

      У Цисиня есть более строгая форма, зачем устраивать испорченый телефон?

      Потому что тут всё спорно. Выживание не является основной потребностью цивилизации, она не обязана постоянно расти и расширяться...


      1. r4nd0m
        02.08.2021 13:31
        +1

        Потому что тут всё спорно.

        То есть, вместо того, чтобы опровергать спорного Цисиня, вы опровергаете свою собственную интерпретацию?


        Выживание не является основной потребностью цивилизации, она не обязана постоянно расти и расширяться

        Технически — да, не обязана. Но те кто не растут и не расширяются или сами деградируют за короткий в космических масштабах промежуток времени, или исчезнут в результате трансформации ближайшей звезды.


        1. Zangasta Автор
          02.08.2021 13:43

          То есть, вместо того, чтобы опровергать спорного Цисиня, вы опровергаете свою собственную интерпретацию?

          Нет.

          Я обсуждаю концепцию "Темного леса" Цысиня в том виде, в котором она присутствует в книге.

          Набор "аксиом" из той же книги, к концепции не относится и только вводит в заблуждение.


          1. r4nd0m
            02.08.2021 13:54

            В том виде, в котором она присутствует в книге, она явно выводится из этого набора аксиом. По-моему, это единственное и главное отличие книги от подобных — какое-никакое обоснование. Иначе бы книга ничем не отличалась от очередной космооперы или мути про попаданцев.


            1. Zangasta Автор
              02.08.2021 14:02

              Иначе бы книга ничем не отличалась от очередной космооперы или мути про попаданцев.

              А она отличается?

              Как по мне --- у Лукьяненко, в дилогии "Звезды - холодные игрушки" больше научных и социальных прогнозов о будущем цивилизаций чем у Лю.


              1. r4nd0m
                02.08.2021 14:23

                А она отличается?

                А вы комментарии читаете? Я вроде бы русским языком написал "По-моему, это единственное и главное отличие книги от подобных".


                у Лукьяненко, в дилогии "Звезды — холодные игрушки" больше научных и социальных прогнозов о будущем цивилизаций чем у Лю

                Мы тут вроде бы не о количестве прогнозов дискутируем.
                Почему вы тогда за Лю взялись, а не за Лукьяненко?
                Ну и:


                Вот ты беседуешь, о какой-то космологической концепции, а оппонент берёт и приводит довод: «А вот в фантастическом романe Лукьяненко...»


  1. Tarakanator
    02.08.2021 14:29

    1)Есть мнение, что и Солнце и Земля особенные с точки зрения пригодности для жизни и эволюции.(если менять параметры, то либо срок жизни системы становится недостаточен для эволюции, либо эволюция(а точнее наступление кислородной катастрофы) замедляется так, что мы просто первые)

    2)Да Солнце звезда 3-го поколения. Вы предлагаете растить жизнь в газовом гиганте? Ну а из чего ежё делать планету у звезды 1-го поколения?

    Я это к тому, что я бы не отвергал гипотезу о том, что мы первые. По крайней мере в некоторой области.


  1. s_a_p
    02.08.2021 15:53
    +1

    Участь злых цивилизаций незавидна. Начав с геноцида, они быстро сталкиваются с множеством проблем — потратив ресурсы на уничтожение соседей, они ничего не получают взамен.

    Только при условии, что уничтожение требует множество ресурсов. В романе явно указано, что это не так, более того, неоднократно подчёркивается, что цивилизации уничтожаются наиболее дешёвым методом.

    Появление прямого и явного злодея Гитлера привело к союзу двух непримиримых конкурентов: СССР и США, которые после победы над злом организуют ООН.

    Это потому, что Гитлер немножко не успел с созданием оружия массового поражения. Если бы немецкие учёные успели создать атомную бомбу, мир был бы совершенно другим. И именно наличие ядерного оружия у СССР и США изменило правила игры.

    Но в масштабах космоса правила бы изменились ещё раз, потому что там, в отличие от земных условий, использование оружия массового поражения безопасно для использующей его цивилизации. А значит нет никаких причин ограничивать его применение. Добавим сюда его дешевизну, и вывод становится очевиден: даже не «злодейская» космическая цивилизация, обладающая оружием массового поражения, будет применять его на всех обнаруженных цивилизациях, потому что есть риск наткнуться на «злодейскую» и самой стать уничтоженной. Таким образом, концепция тёмного леса становится неизбежной с момента появления хотя бы у одной цивилизации оружия, способного уничтожить другую цивилизацию.


    1. Zangasta Автор
      02.08.2021 16:06

      Если бы немецкие учёные успели создать атомную бомбу, мир был бы совершенно другим.

      Именно поэтому Гитлер и проиграл. Его бомбу создавали немецкие ученые --- которые не могли не проиграть сборной команде из европейских и американских ученых.

      Сотрудничество рулит.


      1. s_a_p
        02.08.2021 16:32
        +1

        Делать такие выводы на основании одного нереплицируемого случая — это, во-первых, ошибка выжившего, во-вторых, нарративное искажение. Нет никаких оснований полагать, что при большом количестве повторений (какое может иметь место в космических масштабах) условный инопланетный фюрер будет проигрывать раз за разом. Кроме того, даже если в большинстве случаев это действительно будет так, достаточно одного случая, когда инопланетный фюрер одержит победу — с момента выхода его цивилизации в космос и начнётся тёмный лес.


      1. r4nd0m
        02.08.2021 18:09
        +2

        Гитлер прежде всего проиграл, потому что был психом, отрицающим нормальную науку, а немецкие учёные потом ещё учили эти сборные команды, как нужно строить ракеты.


        И раз уж мы о Гитлере, то ирония в том, что он (да и Япония) тоже в какой то момент начали действовать по правилам "тёмного леса", пытаясь уничтожить конкурентов до их "технологического взрыва". А Япония вообще проиграла именно вследствии технологического взрыва, который вдобавок оказался ещё и ядерным.


    1. vedenin1980
      02.08.2021 18:48

      Добавим сюда его дешевизну, и вывод становится очевиден: даже не «злодейская» космическая цивилизация, обладающая оружием массового поражения, будет применять его на всех обнаруженных цивилизациях, потому что есть риск наткнуться на «злодейскую» и самой стать уничтоженной.

      Не совсем, смотрите с точки теории игр, мы нашли другую цивилизацию, какие у нас есть вероятностьи:

      1. Цивилизация никогда не станет достаточно сильной чтобы быть союзником или врагом с вероятностью p1 (в этом случае, она бесполезна),
      2. Цивилизация может стать в будущем достаточно сильной чтобы быть союзником или врагом с вероятностью 1-p1,
      3. Если мы уничтожим эту цивилизацию — другие могут это обнаружить и счесть нас агресивными и злыми с вероятностью p2 и если они достаточно сильные — то уничтожить нас с вероятностью p3,
      4. Если мы не уничтожим эту цивилизацию — другие могут это обнаружить и счесть нас слишком слабыми с вероятностью p2.1 и если они достаточно сильные — то уничтожить нас с вероятностью p3.1,
      5. Если мы уничтожим эту цивилизацию, которая могла бы стать в будущем нашим союзником с веростностью p4 *(1-p1) и потом встретим более сильную цивилизацию, от которой она могла бы помочь с вероятностью p5.
      6. Если мы не уничтожим эту цивилизацию, которая могла бы стать в будущем нашим врагом с вероятнотностью p6 * (1-p1),
      7. Вероятностью p7, что наших сил не хватит для уничтожения найденной цивилизации, и она станет врагом и сможет уничтожить нас с вероятностью p8,

      Тогда формула оптимального выбора когда вероятностью получить врага или того что другие цивилизации сочтут не уничтожение слабостью против вероятностью получить союзника, того что другие цивилизации сочтут уничтожение агресивностью и того что попытка уничтожения провалиться и приведет к ответному удару.

      Очевидно, оценки вероятностей будут зависит как от скорости технического прогресса, аргесивности найденной цивилизации, от наличие других цивилизацией в окрестности и от изначальных установок цивилизации, изучающей космос.


      1. r4nd0m
        02.08.2021 22:45

        Теория игр — хороший инструмент, когда нужно посчитать вероятный исход некоторого количества игр при известных начальных условиях. Когда на кону будущее цивилизации, даже мизерная вероятность уничтожения перевесит 99% вероятности дружбы.
        Поэтому или незаметно уничтожай, или прячься и наращивай силу, пока не обнаружили. Или беги, если обнаружили.


        1. Если мы уничтожим эту цивилизацию — другие могут это обнаружить и счесть нас агресивными и злыми

        Или подумать, что мы хотим заставить их так думать. А потом подумать, что мы хотим заставить их думать, что мы хотели заставить их так думать. А потом…
        Это и есть цепочка недоверия, она делает бесполезной любую информацию о намерениях цивилизации.


        1. ksbes
          03.08.2021 11:39

          Когда на кону будущее цивилизации, даже мизерная вероятность уничтожения перевесит 99% вероятности дружбы.

          Теория игр умеет «разруливать» и такие ситуации.
          Проблема в том что тут исходов конфликта больше чем два (например, наиболее вероятный исход, по моему мнению, — взаимное истощение и откат в докосмическую эпоху обеих цивилизаций). Да и злобные инопланетяне — далеко не единственная и не самая вероятная причина гибели цивилизации (по крайней мере — нашей).
          Так что оптимальным вариантом вполне может оказаться — дружба с инопланетянами, но война с какой-нибудь «корпорацией зла» внутри нашей.

          Или подумать, что мы хотим заставить их так думать. А потом подумать, что мы хотим заставить их думать, что мы хотели заставить их так думать


          Ну как-то уничтоженная цивилизация это факт, против которого не попрёшь. Те кто уничтожал — явно «покраснеют» и станут приоритетной целью. Но вообще такие находки делают «тёмный лес» светлее и концепция размывается.

          Да и вообще этот «тёмный лес» явно игнорирует тот исторический факт, что война — это не уничтожение, а своеобразный способ общения. Т.е. война — она чаще создаёт цивилизации, чем разрушает их.
          Иными словами — кто сказал, что две цивилизации не сольются в страстном кровавом экстазе в одну? Ну как русские и чукчи после русско-чукотской войны (русские проиграли)?


          1. r4nd0m
            03.08.2021 12:28

            Так что оптимальным вариантом вполне может оказаться — дружба с инопланетянами, но война с какой-нибудь «корпорацией зла» внутри нашей.

            Как вы себе представляете дружбу с инопланетянами без дружбы внутри цивилизации?
            Да, дружба и сотрудничество вполне может быть оптимальным вариантом. Но только если обе цивилизации "мирные" и не знают про "тёмный лес".


            Да и вообще этот «тёмный лес» явно игнорирует тот исторический факт, что война — это не уничтожение, а своеобразный способ общения.

            "Тёмный лес" это космологическая теория. Естественно, что он игнорирует отдельные (и к тому же очень спорные) факты из короткой истории одного вида полуразумных существ, даже в космос толком не вышедших.


            Т.е. война — она чаще создаёт цивилизации, чем разрушает их.

            Какие, по вашему, цивилизации создала война? Не ради завоевания новых территорий и ресурсов, а именно настоящая война, которая на уничтожение?


            В замкнутой системе наверно как то можно использовать войну (или скорее военные действия), как своеобразный способ общения, политики, дипломатии, но даже на Земле люди были готовы пойти на полное гарантированное уничтожение себя и противника. Представляете на что бы они могли пойти, если бы были уверены в отсутствии потерь?


            Иными словами — кто сказал, что две цивилизации не сольются в страстном кровавом экстазе в одну? Ну как русские и чукчи после русско-чукотской войны (русские проиграли)?

            Чтобы слиться, нужна коммуникация. В космосе она невозможна: из за долгого времени передачи информации она просто становится неактуальной. Например, СССР посылает во все стороны "Мир! Труд! Ленин!". Их получают инопланетяне, посылают ответ, а тут уже ни мира, ни Ленина, ни СССР уже тысячу лет как нет.


            1. ksbes
              03.08.2021 13:45

              Как вы себе представляете дружбу с инопланетянами без дружбы внутри цивилизации?

              Вы серьёзно? Гражданских войн не знаете? В той же Америке — во время своей гражданской войны они весьма дружили с европами! А ведь это только самая крайняя форма внутренней вражды.

              «Тёмный лес» это космологическая теория. Естественно, что он игнорирует…

              А-а-а, хмм. Т.е. она игнорирует смысловую нагрузку тех понятий которыми оперирует?! Там какое-то своё, отдельное определение понятия «война»? Думаю нет. А та смысловая нагрузка этого понятия что есть у нас идёт в основном из истории давней и недавней. Иначе как мы можем её получить?
              Ну а если моё высказывание спорное — так о нём и спорте. Зачем так?

              Какие, по вашему, цивилизации создала война?

              Ну, прочтите краткий обзор на историю Китая, хотя бы (там с десяток примеров, самый известный — война трёх царств).
              Ну и оба американских континента, конечно, с латино-американской цивилизацией.

              Нет были, конечно, войны ну прям совсем на уничтожение (по крайней мере одной стороны) — вроде Рима и Карфагена, но я и не утверждаю, что любая война создаст цивилизацию. Хотя даже Пунические войны — именно они сделали Рим из заштатного города именно цивилизацию. И слияние там тоже было, но мало.

              Чтобы слиться, нужна коммуникация

              Чтобы воевать — тоже. Хотя бы для того чтобы убедиться что попали и что ответка не полетела. Да и просто чтобы узнать что там вообще что-то стоящее есть. Не пулять же по всем каменным планетам в потенциально обитаемой зоне! Так никакой пулялки не хватит, да и выдать себя легко можно.
              Ну а лаг по времени — не неразрешимая проблема. Тут своя теория игр начинается — как не прогадать в обмене полезной информацией и ресурсами.


              1. r4nd0m
                03.08.2021 14:08

                В той же Америке — во время своей гражданской войны они весьма дружили с европами!

                Европы и Америки принадлежали к одному биологическому виду.


                Т.е. она игнорирует смысловую нагрузку тех понятий которыми оперирует?!

                Она не оперирует понятиями истории одного конкретного вида существ.


                Там какое-то своё, отдельное определение понятия «война»?

                Там, если я не ошибаюсь, нет понятия войны, как процесса. Там просто уничтожение. Воевать в космосе затратно и чревато раскрытием.


                Ну, прочтите краткий обзор на историю Китая

                Спасибо, прочту как нибудь. Но это опять же "цивилизация" по человеческим меркам.


                Ну и оба американских континента, конечно, с латино-американской цивилизацией.

                Такой цивилизации нет. А вот цивилизации майя, ацтеков и инков эти войны уничтожили.


                именно они сделали Рим из заштатного города именно цивилизацию

                Если бы Рим только воевал, это не сделало бы его цивилизацией, как не сделало Золотую Орду. Рим много скопировал у Греции и добавил немного от себя.


                Чтобы воевать — тоже. Хотя бы для того чтобы убедиться что попали и что ответка не полетела.

                Для этого не нужно обмениваться информацией с врагом.


                Не пулять же по всем каменным планетам в потенциально обитаемой зоне! Так никакой пулялки не хватит, да и выдать себя легко можно.

                Так и не пуляют, ждут пока кто-нибудь себя выдаст.


                Тут своя теория игр начинается — как не прогадать в обмене полезной информацией и ресурсами.

                Теория основана на том, что обмен информации ненадёжен, а ресурсами — нет смысла (космос более-менее однородный).


        1. Lexxnech
          03.08.2021 11:42

          Это работает только если вы уверены что всегда будете уничтожать любого оппонента одним ударом, никаких дипломатических связей у оппонентов нет и ответки гарантированно можно не бояться. Недостаточно считать что дипломатия рискована, нужно быть уверенным что отказ от дипломатии менее рискован.


          1. r4nd0m
            03.08.2021 12:08

            Самое страшное, что может случиться в "тёмном лесу", это раскрытие своего положения. Поэтому, если уничтожение оппонента не гарантировано или есть риск раскрытия, то уничтожение оппонента это самоубийство с высокой степенью вероятности.
            Попытки дипломатии в такой системе технически возможны, например, через анонимные сообщения. Но кто, в режиме 100% паранойи, будет верить таким сообщениям?


            1. Zangasta Автор
              03.08.2021 12:20

              Самое страшное, что может случиться в "тёмном лесу", это раскрытие своего положения.

              Это при том, что в нашей галактике всего 400 миллиардов звёзд. И сверхцивилизация оказывается, не может их все проверить.


              1. r4nd0m
                03.08.2021 12:33

                Как вы собираетесь проверять хотя бы эти 400 миллиардов звёзд?
                Косвенными методами — сложно, и всегда может найтись другая сверхцивилизация, которая научится не отсвечивать на расстоянии.
                Зонды фон Неймана? Так им надо информацию назад как-то передать, а это угроза раскрытия положения.


                1. Zangasta Автор
                  03.08.2021 13:04
                  +1

                  Зонды фон Неймана? Так им надо информацию назад как-то передать, а это угроза раскрытия положения.

                  Информацию не нужно передавать обратно. Информацию о найденной цивилизации можно передать ВО ВСЕ СТОРОНЫ СРАЗУ.

                  Наша галактика очень жиденький "темный лес" --- в котором заведомо невозможно спрятать планету.


                  1. r4nd0m
                    03.08.2021 13:29

                    Можно, и даже наверно проще будет засветить цивилизацию для всех, а там её уже кто-нибудь уничтожит. В книге был такой пример.
                    Но тогда нужно будет как-то защититься от своих же зондов, чем нибудь вроде системы свой-чужой. А это уже, пускай и частичное, но раскрытие информации о себе.


                    1. Zangasta Автор
                      03.08.2021 15:15

                      Можно, и даже наверно проще будет засветить цивилизацию для всех, а там её уже кто-нибудь уничтожит.

                      Тогда нужно будет послать сигналы СО ВСЕХ звездных систем. Заслать туда зонды, имитирующие активность цивилизации. Тогда правильный сигнал не будет виден на фоне миллиардов ложных сигналов.

                      Космос в Темном Лесу будет каким угодно, то только не безмолвным.


                      1. r4nd0m
                        03.08.2021 16:26
                        +1

                        Зачем СО ВСЕХ звездных систем?


                        1. Посылаем зонды.
                        2. Зонды летают по галактике.
                        3. Находят цивилизацию.
                        4. Проверяют — не та ли эта цивилизация, что их послала.
                        5. Если не та — посылают координаты во все стороны.
                        6. Ближайшая цивилизация, достаточно развитая для нанесения незаметного удара, гарантирующего уничтожение, наносит такой удар.

                        Но это работать не будет, потому что зная координаты и время уничтожения можно сильно сократить время поиска цивилизации, которая нанесла удар.


                        Получается, что нанести удар незаметно может только цивилизация, пославшая зонд, при условии, что только она получила сигнал об обнаружении. Но это значит, что зонды в любом случае должны знать, куда посылать сигнал, а это уже опасность раскрытия.


                        Наверно поэтому мы не встретили зондов и космос безмолвный.


                      1. Tsimur_S
                        03.08.2021 17:45
                        +1

                        Но это значит, что зонды в любом случае должны знать, куда посылать сигнал, а это уже опасность раскрытия.

                        а) можно отправлять сигналы с зашифрованными координатами. При шифровании методом типа одноразового блокнота они принципиально невзламываемы. Правда можно восстановить точку отправления пеленгом сигнала. Частично можно нивелировать задержкой отправки сигнала.

                        б) Роутим сигнал через известные ноды флота зондов, на манер сети тора. Сигнал на всю округу будет отправлять выходная нода которая находится далеко от места событий. Они все равно будут пересекаться друг с другом, так что могут обмениваться своими будущими точками маршрута.

                        в) Зонды вместо отправки сигнала проводят экстерминатус.

                        д) Другие зонды, более современные, летают, слушают сигналы зондов разведчиков и производят экстерминатус.

                        е) произвольное сочетание предыдущих пунктов.


                      1. r4nd0m
                        03.08.2021 19:27

                        а) можно отправлять сигналы с зашифрованными координатами. При шифровании методом типа одноразового блокнота они принципиально невзламываемы. Правда можно восстановить точку отправления пеленгом сигнала.

                        Можно. Но обитатели системы поймут, что их нашли, уже по наличию чужого зашифрованного сигнала огромной мощности. И начнут реагировать.
                        И другие цивилизации тоже смогут сообразить: "так, сначала из этой точки был послан зашифрованный сигнал, а через относительно небольшое время система недалеко от точки перестала существовать. АГА!". И начать поиск в сфере радиусом с*t/2 от системы, или с запасом.


                        Частично можно нивелировать задержкой отправки сигнала.

                        Опасно, технологический взрыв же может случиться.


                        б) Роутим сигнал через известные ноды флота зондов, на манер сети тора. Сигнал на всю округу будет отправлять выходная нода которая находится далеко от места событий. Они все равно будут пересекаться друг с другом, так что могут обмениваться своими будущими точками маршрута.

                        Интересно, но огромная задержка (технологический взрыв!). Плюс раскрытие факта существования высокоразвитой цивилизации: "есть сеть — значит по близости где-то цивилизация. Надо поискать."


                        в) Зонды вместо отправки сигнала проводят экстерминатус.
                        д) Другие зонды, более современные, летают, слушают сигналы зондов разведчиков и производят экстерминатус.

                        Будет работать против слабых цивилизаций. При одинаковом уровне развития зонды по определению слабее целой цивилизации. И нужна система свой-чужой, а это возможность раскрытия при попадании на более развитую цивилизацию.


                      1. Tsimur_S
                        03.08.2021 19:53

                        Но обитатели системы поймут, что их нашли, уже по наличию чужого зашифрованного сигнала огромной мощности. 

                        Это если они способны поймать этот сигнал. Тогда их уничтожение возможно будет процессом нетривиальным.

                        Опасно, технологический взрыв же может случиться.

                        Ну с таким подходом любое ожидание опасно. Даже просто когда сигнал дойдет от зонда до "аванпоста" владельцев уже может случится технологический взрыв.

                        есть сеть — значит по близости где-то цивилизация. Надо поискать

                        А само по себе наличие зондов летающих по всей галактике недостаточно раскрывает?)

                        При одинаковом уровне развития зонды по определению слабее целой цивилизации. 

                        При одинаковом уровне развития экстерминатус другой звездной системы втихую вообще невозможен. Если конечно в будущем "меч" не имеет непреодолимого превосходства над щитом.

                        И нужна система свой-чужой, а это возможность раскрытия при попадании на более развитую цивилизацию.

                        Так она же нужна и в вашем плане.

                        Проверяют — не та ли эта цивилизация, что их послала.

                        Если уж не сдерживать полет мысли, то если условная сверхцивилизация озаботится экспансией других видов она может стерилизовать всю галактику(как мы стерилизуем банки перед закаткой), простимулировав серию гамма-всплесков, без всяких разведчиков и темных лесов. Потом и зондами разберет звезды на части, что-бы остановить их распыление в никуда. Выжившие остатки лесных жителей будут штабелированы вместе со своими светилами.

                        Это уже будет не лесник темного леса а лесозаготовитель.


            1. Lexxnech
              03.08.2021 12:58

              если уничтожение оппонента не гарантировано или есть риск раскрытия, то уничтожение оппонента это самоубийство с высокой степенью вероятности.

              Вот только из этого следует противоречивость «темного леса». Потому что если лес темный, то никто не может быть уверен в победе «одним решительным ударом», а значит нападать первому крайне рискованно. И нужно привыкать к жизни, когда вокруг эта проклятая неопределенность, и нет информации о том, как поступят окружающие и как и на Земле необходимо принимать риски вместо того что бы превентивно отстреливать каждую потенциальную угрозу. Ну если не брать магически навязанные условия что никакая колонизация и системы мертвой руки невозможны, а вот межзвездное оружие уничтожающее цивилизации одним ударом при этом вполне доступно всем и каждому.
              В этом случае дипломатия может снижать риски, поскольку снижает неопределенность. Да, есть риск что контакт перерастет в конфликт, но и отсутствие контакта тоже может привести к конфликту.
              У цивилизации даже нет гарантий, что ее уже не обнаружили и не нападают только потому, что не хотят рисковать поскольку не уверены в победе.


              1. r4nd0m
                03.08.2021 13:42

                Я не спорю, что теория противоречивая. Но при этом оригинальная и интересная.
                Тут, как с уравнением Дрейка — слишком много неизвестных. Вероятность обнаружения, например, прямо зависит от решения уравнения Дрейка.


                Вот вы подразумеваете, что одним ударом можно не уничтожить. А в книге было оружие измерений, которое гарантированно уничтожало всё. И вообще, в книге Земле повезло попасть на инопланетян, которые общались и физически летели воевать только потому, что у них своя планетарная система нестабильная была. Хотели бы уничтожить Землю — послали бы втихаря одну "каплю", которая там за считанные минуты уничтожила весь земной космофлот.


                1. Lexxnech
                  03.08.2021 14:37

                  Я не случайно упомянул невозможность колонизации. Что бы гарантировано уничтожить все, нужно сперва точно знать расположение всех колоний в других системах. А лес, напоминаю, темный, и обнаружить заселенные системы сложно, иначе сама идея прятаться и бить исподтишка работать не будет. При этом, по условиям у нас цивилизации уничтожения боятся, и поэтому будут крайне заинтересованы в расселении, как раз для того что бы их нельзя было прихлопнуть одним ударом.
                  И возникнет глупая ситуация, когда несколько цивилизаций столетиями знают о существовании других, знают расположение какой-то части их систем, нападать первыми боятся и усиленно делают вид что не замечают друг друга. В таких условиях дипломатия может наоборот снизить риски.


                  1. r4nd0m
                    03.08.2021 16:36
                    +1

                    Какая будет цель у дипломатии в таких условиях? Обезопасить себя от нападения? Уже достигнуто.
                    Раздел сфер влияния? Это раскрытие положения колоний.


                    Мне кажется, дипломатия — это артефакт культуры человечества, которое более-менее ценит жизнь индивида. Попадёшь на зергов — и как с ними договариваться? Ну пожертвуют пару десятков колоний, зато вычислят все колонии противника и уничтожат их.


                    1. Lexxnech
                      03.08.2021 17:53

                      Ну во первых для получения более предсказуемой ситуации. Все таки все время действовать исходя из того что все о нас вероятно знают но это не точно, и не совершенно неясно сколько и чего именно знают, немного неудобно. Не обязательно раскрывать все, что бы уменьшить риск конфликтов там, где их можно избежать. Как минимум те действия для которых нежелательна неожиданность для окружающих, не будут ими. Ну и для частичного разделения сфер влияния, в тех ситуациях когда это снижает риски и не предоставляет лишней информации, например в формализации определения кому будет принадлежать система если обе цивилизации решили ее занять с небольшой разницей по времени.
                      Во вторых, если отношения уже установлены и война уже не идет, то возможно и всякое на тему культурного обмена, в тех случаях когда он возможен. С зергами не получится, а вот с другими индивидами вполне. Отсутствие контакта же гипотеза темного леса выводит не из имперской религиозной ненависти к любой ксеносской скверне а из страха уничтожения. Это не приводит к прямому снижению рисков (хотя может к привести к косвенному), но если обмениваться уже можно, то почему нет? Если контакт установлен то все, можно выдохнуть и попытаться вспомнить что взаимодействие это не обязательно попытка нагнуть друг друга.


  1. name_taken
    03.08.2021 22:24
    +1

    Проиграв несколько сценариев взаимодействия цивилизаций, я пришел к выводу, что зло всегда уничтожает само себя. Довольно банальный вывод, постоянно воспроизводящий себя в сотнях конфликтов по всему миру.

    А можно другой вывод сделать. Например что историю пишут победители, и именно поэтому «зло» всегда проигрывает.


  1. Lexxnech
    04.08.2021 11:00

    Я тут подумал на тем что обсуждал выше и обнаружил, что в гипотезе есть нехилая такая подмена, которая как раз вылезает при добавлении в простую модель усложняющих деталей вроде колонизации.
    Вот в том месте гипотезы, где сравниваются риски различных стратегий, на самом деле сравниваются нифига не стратегии. На одной стороне у нас действительно стратегия, с возможными исходами различной успешности. А с другой стороны мы берем не стратегию а уже достигнутый успешный результат. В одну половину таблицы мы записываем все возможные исхода, а в другую только те, которые нас устраивают. Сравнивается не дипломатия с войной, а дипломатия с победой в войне. Соответственно что бы сравнение работало, нам необходимо постулировать что инициатор конфликта всегда побеждает.
    То есть у дипломатия «не работает» не потому что расстояния делают ее ненадежной. Дипломатия и не бывает надежной, нарушения договоров, ложь, нападения без объявления войны и прочие подставы на Земле происходили и происходя. Но скажем США с СССР предпочли в холодную войну дипломатию войне совсем не потому что могли друг-другу доверять. Просто все понимали, что война тоже не обязательно будет маленькой и победоносной.
    В книге это можно обеспечить авторским произволом. Поскольку автор сам определяет, что в его вселенной возможно а что нет, то работать будет вообще любой набор предположений. Если автор этого хочет, то и негуманные цивилизации будут гарантированно самоуничтожатся, и мирные цивилизации гарантированно проигрывать и все цивилизации будут отказываться от экспансии в пользу виртуальной реальности и т.д и т.п.
    Но нужно понимать, что если требуемое для ее работы условия не выполняются в реальном мире, то не будет работать и гипотеза. Если не выполняется правило «атакующий всегда побеждает и выживает при этом» то гипотеза темного леса не верна.


  1. gans2
    09.08.2021 13:18

    Как и Лю Цисинь, ТС игнорирует возможности гравитационных линз звезд. Оба про них не подозревают.

    Прятаться уже поздно. Нас видно и слышно во все гравископы и рефрактоскопы в Известном Космосе Ларри Нивена (пространство вокруг Солнца, диаметром около 60 световых лет ). Отчетливо слышно. И все наши ужимки и прыжки видно. Если есть кому смотреть.


  1. JPEGEC
    22.08.2021 07:58

    Без обид, но текст пустой, не раскрывает поднятый вопрос, не делает никакого анализа и прогнозирования.

    Просто некое "автору так кажется" и "коллега Иван не смог" которое видимо должно что-то значить.

    Возьмите не муравьев, а примерно равных конкурентов, там такие страсти кипят в дружных коллективах что диву даешся. Оболгут, подсидят, морально уничтожат и все ради всего лишь чуть более высокого места в иерархии. Что будет если речь зайдет про жизнь и выживание?